Está en la página 1de 91

1.

En pacientes sin antecedentes patologicos conocido es visto el edema en las


extremidades inferiores luego de largos periodos de bipedestacion, identificado por la
presencia del signo de godete o fovea, sin cambios de color ni signos de inflamacion.
¿Cuál de los siguientes mecanismos es mejor explicar este fenómeno? Aumento de la
presión hidrostática

2. Masculino de 65 años con postquirúrgico de 1 semana de evolución por hernia


umbilical y vólvulo en íleon. Refiere dolor e hinchazón de la pierna derecha. El
cirujano sospecha de un trastorno vascular e indica un Doppler que muestra trombosis
de las venas profundas de la pierna. Cuál de los siguientes mecanismos es más
probable que contribuyo a esta condición? Inmovilización

3. Una niña de 6 años tiene erupción con manchas de color marron rojizo en su cara,
tronco y extremidades proximales de 3 dias de evolucion. En el examen físico, ella
tiene lesiones ulceradas de 0.2 cm a 0.5 cm en la mucosa de la cavidad oral (Koplik) y
linfadenopatía generalizada. Al 4to dia presenta tos con poco esputo que empeora
progresivamente. ¿Cuál de los siguientes virus es más probable que produzca estos
hallazgos? Sarampión

4. Masc. de 37 años, acude a la consulta por disminución de la fuerza y sensación de


dolor con ardor de muñeca derecha, también refiere cólicos de intensidad moderada
en la historia refiere que trabaja de bomba de gasolina desde hace 10 años. ¿A cuál
concepto más probable estuvo expuesto? Plomo

5. Masc de 34 años con antecedentes de endocarditis bacteriana presenta hemorragias de


1 mm alrededor de la órbita de sus ojos (mostrados en la imagen). Cuál es el
apropiado término usado para describir esta forma de hemorragia superficial?
Petequias

6. Fem. de 22 años de edad desarrolla una cara roja después de que se le haga una
pregunta durante la clase. ¿Cuál de las siguientes afirmaciones describe mejor esta
reacción vascular? Hiperemia activa

7. Masc. de 59 años, experimenta un dolor torácico intenso repentino que irradia a su


espalda. En el examen físico su presión sanguínea es 170/110 mm Hg. Se realiza una
pericardiocentesis y se extrae sangre. ¿Cuál de los siguientes hallazgos patológicos
muy probablemente ocurrió en su aorta? Aneurisma disecante

8. Femenina de 35 años padece de síndrome antifosfolipídico; a causa de esto desarrolla


trombos en circulación arterial y venosa. Uno de estos trombos ha alcanzado la arteria
renal y obstruye el aporte sanguíneo. ¿Cuál de los siguientes fenómenos en la
evolución del trombo explica el evento clínico? Embolización

9. Fem. de 40 años muere después de una larga historia de una enfermedad caracterizada
por disnea, ortopnea, hepatomegalia, distensión de las venas del cuello y edema
periférico. La superficie de corte del hígado tiene aspecto de Nuñez moscada. ¿Cuál
de los siguientes trastornos es la causa más probable de estos hallazgos? Insuficiencia
cardiaca derecha

10. Masculino de 70 años de edad, que vivió 60 años en una aldea de nativos africanos
(sub- saharan), acude al médico por cuadro clínico de ascitis, trastorno la función
hepática, elevación de las enzimas hepáticas, sin anticuerpos para virus de hepatitis C
ni B niega consumo de alcohol. Se le diagnostica con carcinoma hepatocelular el
paciente tuvo exposición a un agente carcinogénico en su región natal cuál casino.
¿Cual carcinogénico está vinculado al desarrollo de este cáncer? Aflatoxina B1

11. Pertenece al tipo II, de hipersensibilidad tipo citotóxico: anemia hemolítica


autoinmune

12. La infección por genotipos de alto riesgo de virus papiloma humano está relacionada
con el carcinoma de células escamosas por degradación de genes supresores
tumorales ¿Cuál de los siguientes mecanismos explicaría mejor esta asociación?
OncoProteínas Virales E6-E7

13. Masculino de 7 años de edad presenta lesión de color café oscuro 8 mm de diámetro,
de bordes irregulares, dolorosa en la pared del tórax, se realiza biopsia que reporta un
melanoma y antecedentes de recibir tratamiento para un carcinoma de células
escamosas hace 1 ano. Este paciente tiene la condición de origen hereditario que su
doctor le explica que es un factor predisponente para desarrollar este tipo de cáncer de
las siguientes: ¿ Cuál cree usted que es la enfermedad genética que más
probablemente tenga este paciente? Xeroderma pigmentoso
14. Masc. de 28 años con múltiples lesiones osteolíticas en cráneo y columna vertebral.
Calcificaciones en los túbulos renales. Es diagnosticado con mieloma múltiple. El
calcio sérico es (8,5 a 10,2). La anormalidad metabólica descrita aquí es
probablemente debido a la sustancia? factor de activación osteoclastos

15. Una mujer de 54 años de edad que ha sido diagnosticada con cáncer de mama en
etapa temprana se somete a una cirugía para una tumorectomía para extirpar un
pequeño tumor decretado por monografía. El informe de patología confirma la etapa
temprana del cáncer, comentarios adicionales sobre el hecho de que hay desmoplasia
significativa en el tejido circundante. el término desmoplasia se refiere a:
proliferación de tejido conectivo fibroso no neoplásico

16. Paciente masculino de 52 años, fumador de tabaco y cigarrillo, presenta carcinoma de


células pequeñas en pulmón izquierdo con metástasis de diversos órganos. Cuál de las
siguientes cree usted que fue la vía inicial más común para metástasis de cáncer?
Linfática

17. Masculino de 34 años de edad, fumador, consulta por presentar ascitis de 3 meses de
evolución y dificultad respiratoria. al examen físico, presenta taquipnea
Hepatomegalia. Luego de múltiples estudios se le realiza biopsia peritoneal en un
tumor seroso con abundante democracia y proliferación vascular haciéndose
diagnóstico De mesotelioma bien diferenciado. ¿Cuál de los siguientes marcadores
tumorales nos ayudaría a establecer el diagnóstico de esta neoplasia? Cytokeratins
CK5/6

18. Femenina de 16 años de edad de raza negra quien presenta eritema facial Como alas
de mariposas e inflamación de articulaciones es referida consulta de reumatología por
diagnóstico de lupus eritematoso sistémico. Diga a cuál tipo de hipersensibilidad
pertenece: Tipo III

19. Femenina de 36 años que refiere haber sido diagnosticada con una inmunodeficiencia
congénita a la edad de 8 años pero que siempre ha estado asintomática. ahora presenta
procesos alérgicos y recurrentes y sindrome de malabsorcion desde hace 4 meses.
¿Cuál de las siguientes es el diagnóstico más probable? Deficiencia selectiva de IgA

20. Masculino de 78 años con una historia de fumador de 6000 paquetes de cigarrillos por
año, se presenta a la emergencia con dificultad para respirar y hemoptisis. Radiografía
de tórax muestra una gran masa centrada en el interior del campo pulmonar izquierdo.
El calcio sérico es de 13, 0 mg/dl ( normal 8,5 a 10,2). la anormalidad metabólica
descrita aquí es probablemente debido a la elaboración de qué sustancia? Hormona
relacionada con la paratiroides

21. Masculino de 44 años diagnosticado con glomerulonefritis membranosa por la cual


recibió trasplante renal izquierdo como tratamiento definitivo, elegante fue uno de sus
tres hijos. acude a consulta de nefrología por presentar cuadro clínico de fatiga y
aumento de urea y creatinina luego de 3 semanas del trasplante. la biopsia renal
mostró infiltrado inflamatorio expensas de linfocitos, macrófagos y necrosis cortical.
de acuerdo de sus hallazgos clínicos y patológicos, diga cuál tipo de rechazo de
trasplante se refiere el caso. Agudo

22. Masculino de 64 años de edad tiene una larga historia de cáncer de prostata
metastasico en el momento del diagnóstico. Durante los últimos dos meses, ha tenido
pérdida significativa de peso, pérdida de apetito y pérdida de energía. su actual
condición de caquexia se puede atribuir a cuál de los siguientes? factor de necrosis
tumoral-a

23. Femenino de 46 años de edad, diabética, la cual presenta como complicación una
enfermedad renal crónica por nefropatía membranosa recibiendo trasplante renal de su
esposo. dos meses tras el trasplante paciente presenta fiebre alta y oliguria de un día
de evolución, con Resultado de creatinina 5.0mg/Dl y urea en 200 mg/Dl. Se realiza
biopsia de riñón trasplantado que muestra inflamación intestinal con infiltración de
macrófagos y linfocitos, edema, tubulitis intersticial, necrosis tubular y fibrinosa y
daño vascular. ¿De acuerdo a estos hallazgos histológicos Qué tipo de rechazo a
trasplante presenta Y por qué mecanismo se produjo? Rechazo agudo, ambos
mecanismos humoral y celular

24. Una mujer de 46 años que presenta una masa irregular de 3 cm que no es móvil. Se
realiza una mastectomía y el patólogo en la sección de la mamá encuentra una masa
de 3x3,5cm que no tiene bordes discretos, pero parece infiltrarse en el estroma
mamario circundante. la masa es firme, blanca y tiene consistencia fibrosa. ¿ cuándo
las siguientes características se demuestran con mayor probabilidad por la aparición
de esta masa? Desmoplasia

25. Es correcto sobre los oncogenes: versiones mutadas que funcionan de manera
autónoma, habiendo perdido la dependencia de señales normales que promueven
el crecimiento.

26. Fem. 80 años presenta pérdida de sangre en heces, Refiere que en su familia hay
historia de pólipos. En la colonoscopia se observa masa tumoral en su colon
ascendente, se diagnostica con adenocarcinoma. ¿ Cuál de las siguientes
antioncogenes ( gen supresor tumoral) es más probable que esté mutado? APC

27. Femenina de 43 años de edad Presenta una masa palpable de 4cms de diámetro en el
cuadrante externo a la mama izquierda, se realiza biopsia y reporta un carcinoma
ductal infiltrante todo lo que se decide mastectomía con disección de los ganglios
linfáticos axilares. ¿Cuál de las siguientes sería el mejor predictor de buen pronóstico
en este paciente? No Metástasis en los ganglios linfáticos
28. Masculino de 13 años, HIV positivo, presenta dificultad respiratoria, hemoptisis, tos y
fiebre alta, su doctor piensa que puede estar cursando con una tuberculosis por lo que
realiza una prueba cutanea de la tuberculina que resulta positiva con un area firme de
color rojo oscuro en el antebrazo, aparece 60 horas despues de la inyeccion del
derivado proteico purificado (PPD). Este hallazgo es más probable que sea una
consecuencia de cuál de los siguientes tipos de reacción de hipersensibilidad?
Hipersensibilidad de tipo IV

29. Masculino de 5 años es llevado a emergencia con edema generalizado,


hipoalbuminemia, albuminuria e hiperlipidemia. No se registra hematuria ni
hipertensión. Refiere la madre que presentó una infección de garganta hace varios
días. En este caso el edema lo más probable esté causado por cuál de los siguientes
mecanismo: Disminución presión oncótica

30. Un masculino de 65 años fue al médico a causa de un infarto de miocardio. las


arterias coronarias eran cordones sólidos, la izquierda con marcada aterosclerosis y
presenta un trombo, que esperaría encontrar en este trombo? líneas de Zahn

31. Masculino 56 años pescador en el río Ozama en las cercanías de dos fábricas de
plásticos y fertilizantes. acude al médico parestesias, ataxia, disartria, pérdida de
audición y disminución de la visión. ¿Cuál de los siguientes agentes químicos
probablemente se asocian con la patogénesis de la enfermedad neurológica de este
hombre? Mercurio

32. Femenina de 23 años se presenta con inicio reciente de flujo vaginal. El examen
físico revela múltiples vesículas claras en su vulva y vagina. un frotis de material
obtenido de una de estas vesículas revela mostrado en la imagen. estos cambios son
probablemente por cuál de los siguientes organismos? virus del herpes simple
(VHS)

33. Femenina de 15 años que presenta fiebre, dolor de garganta y adenopatías cervicales.
luego desarrolla hepatomegalia y esplenomegalia que dura dos meses. En el frotis de
sangre periférica se observan leucocitos con linfocitos atípicos. Cuál de los siguientes
tipos de células tiene más probabilidades de eliminar las células infectadas por el
virus: CD8 citotóxicos
34. Masculino de 70 años de edad, originario del sur del país presenta edema de uno de
sus miembros inferiores caracterizado por no dejar foveola (duro) y edema del
escroto, este cuadro tiene varios meses de evolución. El mecanismo más probable de
este cuadro clínico es: Obstrucción linfática.

35. Masculino de 16 años presenta una lesión en la cara que mide aproximadamente 1.5
cm diagnosticado como carcinoma invasivo de células escamosas. Tiene cutáneas
similares que han ocurrido principalmente en áreas expuestas al sol. es probable que
este paciente tenga un tipo de grupo de enfermedades hereditarias asociadas con ADN
inestable y mayor incidencia de carcinoma. cuál de los siguientes es el diagnóstico
más probable? xeroderma pigmentosa

36. Masculino de 4 años es llevado a emergencias, la madre dice que presentó una
infección de garganta hace varios días. Actualmente presenta edema generalizado,
albuminuria, hiperlipidemia, niega la hematuria y la hipertensión. En este caso el
edema lo más probable esté, causado por cuál de los siguientes mecanismos?:
Disminución de la presión oncótica

37. Femenina de 48 años presenta sangrado vaginal anormal especialmente poscoital. Se


le realiza una biopsia que muestra un carcinoma de células escamosas del cérvix. A
esta muestra se le realiza pruebas moleculares de las siguientes, ¿cuáles son más
probable que ser agente identificado?: Virus del papiloma humano tipo 16 o 18

38. Masculino de 65 años fue al médico a causa de un infarto del miocardio. Las arterias
coronarias eran cordones sólidos a la izquierda con marcada arteriosclerosis y
presenta un trombo, que esperaría encontrar en este?: Líneas de Zahn.

39. Femenino de 40 años muere después de una larga historia de una enfermedad
caracterizada por disnea, ortopnea, hepatomegalia, distensión de las venas del cuello y
edema periférico. La superficie de corte del hígado es mostrada en la imagen ¿cuál de
los siguientes trastornos es la causa más probable de estos hallazgos?: Insuficiencia
cardiaca derecha.

40. Del caso clínico anterior, el mecanismo patológico corresponde a: Congestión pasiva
crónica

41. Moreno de 60 años alcohólico desde los 25 años presenta sangrado GI alto por ruptura
de varices sangrantes esofágicas y fallece. En la autopsia, se descubre que el abdomen
sobresaliente del paciente contiene un gran volumen de líquido seroso, ¿Cuál es el
término apropiado utilizado para describir este fluido?: Ascitis

42. Femenina de 3 años, presentó fiebre, escalofríos, púrpuras generalizadas y rigidez en


el cuello. La madre refiere que no cree en las vacunas y que el niño no recibió
ninguna. El paciente presenta hipotensión, coagulación intravascular diseminada y
fallece dos días después. El examen post mortem demuestra la suprarrenal derecha
como la imagen revela, ¿A cuál de los siguientes eventos corresponde este hallazgo?
Síndrome de waterhouse-friderichsen

43. Un niño de 3 años con retraso en el desarrollo, ataxia, convulsiones y risas


inapropiadas tiene un cariotipo normal (46, XY), pero el análisis de ADN muestra que
ha heredado los dos cromosomas número 15 de su padre. Su trastorno genético es un
ejemplo de: Impresión genética (Genetic imprinting).

44. Un masculino de 46 años sufre un accidente de tránsito recibiendo todo el impacto en


la cabeza, falleció secundario al trauma craneoencefálico. En la biopsia se evidencia
acumulacion excesiva de líquido en la sustancia blanca, ¿cuál de los siguientes es la
causa más probable de muerte de este paciente?: Edema vasogénico.

45. Femenina de 50 años a la cual recientemente le realizaron histerectomía por


leiomiomatosis uterina, 3 días posteriores mientras se encontraba aun acostada
presenta dificultad respiratoria súbita y fallece de un tromboembolismo pulmonar.
¿Cuál es el origen más frecuente de este émbolo?: Venas femorales

46. Es el exceso de sangre en un tejido por incremento en la llegada a través del sistema
arterial: Hiperemia activa.

47. Masculino de 67 años, pescador de la provisión de Guangdong en China presenta


fiebre y cefalea, luego de 4 días desarrolla el síndrome respiratorio agudo severo
(SRAS) se sospecha fue ocasionado por un tipo de coronavirus. En los tejidos de
pulmón hay un daño alveolar difuso, ¿Cuál otro hallazgo usted esperaría encontrar en
las células sincitiales pulmonares?: Células multinucleadas sin inclusiones virales.
48. Recién nacido de 40 semanas presenta microcefalia, hepatoesplenomegalia e ictericia
de 2 semanas de evolución, actualmente presenta disnea marcada y se decide tomar
biopsia de pulmón qué evidencia los siguientes cambios citopáticos que corresponde
a: Citomegalovirus.

49. La característica citopática del virus varicela-zoster es: Inclusiones intranucleares.


50. Masculino de 13 años presenta autismo, retardo mental. Al examen físico presenta
aumento de la circunferencia de la cabeza con cara alargada. Presenta
hiperextensibilidad e hipotonía. Su tío materno está igualmente afectado, cuál de los
siguientes es su diagnóstico? Síndrome de X frágil.

51. Masculino de 16 años presenta síndrome caracterizado por defecto en el tejido


conectivo caracterizado por hiperelasticidad y fragilidad de la piel, hipermovilidad y
dolor de las articulaciones así como mayor riesgo de complicaciones como la
disección aórtica, cual de los siguientes es el diagnóstico más probable?: Sindrome
de Ehlers Danlos.

52. Tres niños de una misma familia presentan historia de múltiples fracturas asociadas a
problemas dentales y problemas de audición. Presentan peso y talla normal para la
edad y sexo. Ambos padres no están afectados. ¿Cual de las siguientes anomalías
genéticas es más probable que pueda explicar los hallazgos en estos niños?
Mosaicismo gonadal.

53. Masculino de 63 años diagnosticado con falla ventricular derecha fue llevado a la
emergencia por dificultad respiratoria secundaria a un edema pulmonar. No pudo ser
estabilizado y falleció. En autopsia se evidencia un hígado congestivo, ¿qué hallazgos
microscópicos espera encontrar en la biopsia hepática? Dilatación de los sinusoides
con atrofia de los hepatocitos centrolobulillares.

54. Como se llama a la capacidad de un microorganismo para causar enfermedad?


Virulencia.

55. Un niño de 10 años presenta xantomas en la superficie extensora de sus antebrazos.


Los estudios de laboratorio demuestran un colesterol total en suero de 820 mg/dL. La
madre y el abuelo materno del niño también tienen colesterol sérico elevado. Es
probable que este paciente tenga una mutación en el gen que codifica cual de las
siguientes proteínas participa en el metabolismo de los lípidos? Receptor de
lipoproteínas de baja densidad.

56. Masculino de 3 meses de edad presenta marcada distensión abdominal. Se realiza


ecografía donde se evidencia importante esplenomegalia y hepatomegalia. 3 meses
después los padres notan que el niño presenta flexión hacia atrás del cuello,
estrabismo y dificultad para abrir la boca (trismo) qué se hace más intenso al paso de
los días. Luego de realizar estudios genéticos paciente fallece a los 2 años y medio de
vida por complicaciones de su enfermedad. ¿Cuál de los siguientes cree usted que era
el diagnóstico más probable? Enfermedad de Gaucher tipo 2.

57. Años de edad el cual es el pronosticado desde su infancia con una enfermedad de
herencia autosómica dominante caracterizada por ser muy alto de estatura con
extremidades largas y dedos extremadamente largos ectopia lentis y mayor riesgo de
enfermedades cardiovasculares Cuál de las siguientes es la causa molecular de esta
enfermedad? Mutación en el Gen que codifica la fibrina 1 en el brazo largo del
cromosoma 15.

58. El virus de Epstein-Barr (EBV) fue el primer virus vinculado en el desarrollo de


neoplasias, cuál de los siguientes cánceres está relacionado con esa infección?
Linfoma de Burkitt

59. Con respecto al virus EBV es conocido de que produce mononucleosis infecciosa que
cursan con adenopatías en el cuello y hepatoesplenomegalia leves, cuál de los
siguientes linfocitos atípicos en la sangre usted encontraría en estos pacientes?
Linfocitos T activados

60. Femenina de 21 años presenta durante 7 días fiebre adenopatías y


hepatoesplenomegalia leve y se sospecha de mononucleosis infecciosa. ¿Cuál prueba
usted indicaría para el diagnóstico y detección de esta infección? Anticuerpo
heterófilo

61. La tríada de Virchow afirma que los siguientes tres factores predisponen a la
trombosis: daño al endotelio, flujo sanguíneo desordenado e hipercoagulabilidad.
Durante la trombogénesis, ¿Cuál de las siguientes sustancias es secretada por las
plaquetas activadas? Tromboxano A2

62. ¿Cuál de los siguientes procesos está vinculado con el infarto cerebral? Necrosis
licuefactiva y gliosis.

63. Femenina de 28 años quien cursa el segundo trimestre de embarazo la misma lleva un
control prenatal cercano ya que en su familia cuenta con 2 miembros con síndrome de
Down. Durante el último estudio sonográfico apareció un engrosamiento del pliegue
nucal, como esperamos encontrar los siguientes marcadores en este caso? Alfa
fetoproteína es baja

64. Masculino de 25 años diagnosticado con trisomía 21 presenta múltiples hematomas


asociado a trombocitopenia, palidez generalizada de la piel evidenciándose en el
hemograma que presenta anemia severa y disminución de los glóbulos blancos.
Tomando en cuenta la enfermedad asociada a este paciente, cuál de las siguientes
enfermedades cree usted que podría ser la causa del cuadro actual? Leucemia aguda

65. Masculino de 40 años presenta debilidad muscular. Se realiza electromiograma que


revela una elección cística en el músculo deltoides. Se realiza biopsia que revela que
la lesión es por cisticercosis. ¿Cuál de los siguientes hallazgos microscópicos guarda
relación con las manifestaciones clínicas de esta enfermedad? Atrofia muscular

66. Femenina de 63 años acude al hospital por presentar náuseas, vómitos, fuerte dolor de
cabeza y fiebre alta cada 3 días y presenta ictericia. Refiere semanas previas estuvo de
visita en un país tropical y se sospecha de malaria. Tras tres días hospitalizada la
paciente muere, en el hígado presenta un pigmento que corresponde a: Hemozoina

67. Femenina de 45 años con hipertensión descontrolada desde hace 3 meses. El último
mes tuvo un cuadro clínico que inició con disnea, tos no productiva, la radiografía de
tórax evidencia las líneas de Berley B. Ella pidió no ser tratada con fármacos y obtuvo
el uso de té de ajo. La última semana la paciente presentó marcada cianosis y
dificultad respiratoria y falleció. El autopsia en pulmón se encontró que corresponde a
Daño alveolar difuso

68. Masculino de 84 años, postrado en cama por Alzheimer, experimenta una aparición
repentina de dolor pleurítico hemoptisis y fallece. En cuál de los siguientes sitios
surgió la lesión subyacente que condujo a esta complicación? Venas de las
extremidades inferiores

69. Padres de un adolescente de 14 años de edad acuden al pediatra preocupados porque


ha presentado lesiones generalizadas en la piel desde hace aproximadamente 6 meses
que van en incremento. Al examen físico estos tumores son suaves, bien delimitados,
del mismo color de la piel y de tamaño variable. Además presentan nódulos
pigmentados en el iris (nódulos de Lisch), el pediatra le explica que su hijo podría
padecer una enfermedad de origen genético, cuál de los siguientes es el diagnóstico
más probable? Síndrome de Von recklinghausen

70. Con relación a la pregunta anterior, ¿Cuál de las siguientes patologías estos pacientes
presentan un mayor riesgo de desarrollar? Leucemia mielocitica

71. Masculino de 52 años, alcohólico, es llevado a la emergencia por presentar sangrado


en las encías y múltiples hematomas. En el examen físico presenta hepatomegalia y
estaba pálido. El hemograma tenía anemia con conteo bajo de plaquetas ¿Cuál de las
siguientes enfermedades cree usted que sea el diagnóstico más probable en este
paciente? Hipertensión portal

72. Masculino de 17 años es llevado a la emergencia por presentar convulsiones. Al


examen físico presenta frecuencia cardíaca con 96 latidos con un ritmo irregular
evidenciado en electrocardiograma y una temperatura de 41 grados. Se realiza una
tomografía de cráneo donde es evidente una hemorragia subaracnoidea. De las
siguientes sustancias ¿Cuál sería la respuesta del estado clínico en este paciente?
Anfetaminas

73. Femenina de 14 años con anemia falciforme acude a urgencias porque está
experimentando dolor óseo intenso y crisis vasooclusiva. Una tomografía
computarizada abdominal muestra evidencia, imágenes en cuña a nivel del vaso. La
esplenectomía revela la imagen, ¿cómo clasificarías estas lesiones? Infartos
anémicos blancos

74. Masculino de 23 años que trabaja de motoconcho se sometió a una cirugía para
fractura de pelvis y fémur izquierdo resultante de un accidente. Al día siguiente
desarrolla disnea y dificultades del habla, ¿cuál de los siguientes tipos de embolia es
la causa más probable? Grasa
75. Embarazada de 38 semanas de gestación acude a la emergencia por presentar
contracciones uterinas y sangrado vaginal por lo que se realiza una cesárea
obteniéndose una niña sana pero con un peso de 2.1 kg, cuál de los siguientes es el
mayor factor de riesgo para bajo peso al nacer? Fumar cigarrillos

76. Con relación a la pregunta anterior ¿Cuál de los siguientes podría ser una
complicación en esa niña? Mayor riesgo de desarrollar otitis media

77. Masculino de 57 años fumador de 10 paquetes de cigarros al día presenta


complicaciones de hipertensión arterial y bronquitis crónica aguda el doctor por un
fuerte dolor en epigastrio acompañado de vómitos, su médico le recomienda dejar de
fumar ya que el cigarro está asociado a muchas otras enfermedades de las siguiente.
¿Cuál crees usted que podría estar desarrollando a nivel gastrointestinal este paciente?
Úlcera péptica

78. Una mujer de 33 años se presenta con inicio y recientes de flujo vaginal, el examen
físico revela múltiples vesículas claras en su vulva y vagina, un frotis de material
obtenido de una de estas vesículas revela lo mostrado en la imagen, estos cambios son
probablemente por cuál de los siguientes organismos? Virus del herpes simple
(VHS).

79. Una mujer de 35 años de edad padece de síndrome antifosfolipídico, a causa de esto
desarrolla trombos en circulación arterial y venosa, uno de estos trombos alcanzado la
arteria renal y causa que se obstruye el aporte sanguíneo. ¿A cuál de los siguientes
fenómenos en la evolución del trombo se debe este último evento? Embolización

80. Masculino de 48 años el cual trabaja hace 15 años en una fábrica de baterías es
llevado a la emergencia por presentar tos con expectoración sanguinolenta y dificultad
respiratoria severa, además refiere que en los últimos días ha estado orinando poco
evidenciándose proteinuria en el examen de orina. Días después el paciente fallece. Se
realiza autopsia que revela enfisema pulmonar además de daño tubular y glomerular a
nivel renal. Se confirma en sangre una sustancia que es responsable de sus hallazgos
según la historia y los hallazgos de autopsia. ¿Cuál cree usted que fuera la sustancia
identificada en la sangre de este paciente? Cadmio
81. ¿Cuál de los siguientes no predisponen al tromboembolismo pulmonar? Hemofilia

82. Una mujer infértil fue examinada genéticamente y se encontró que tenía deleciones de
porciones del brazo largo y brazo corto del cromosoma X lo que resulta en un
cromosoma en anillo 46, X, r (X). Cuál de los siguientes es el diagnóstico más
probable? Síndrome de Turner

83. Una mujer sexualmente activa de 28 años se somete a un examen de rutina el examen
pélvico no revela anormalidades se obtiene una prueba de Papanicolaou el informe de
citopatología indica la presencia de células como muestra en la imagen HPV

84. Recién nacido de 37 semanas con anemia severa e hidropesía fetal Parvovirus b19

85. Femenina 2 años con neumonía RSV

86. Niña de un mes y 15 días hija de padres sanos parto de término eutócico y sin
antecedentes mórbidos neonatales. Inicia síntomas respiratorios en ausencia de fiebre
caracterizado por tos paroxística con cianosis peribucal y obstrucción bronquial leve.
A su ingreso al servicio de urgencias destacaba ausencia de compromiso séptico
afebril dificultad respiratoria moderada con una frecuencia respiratoria la biopsia de
pulmón muestra: CMV
87. Un hombre de 60 años de edad con un historial de múltiples infartos de miocardio es
hospitalizado por falta de aire. El examen físico revela marcada distensión yugular,
hepatomegalia, ascitis y edema con fóvea. Una radiografía de tórax muestra
cardiomegalia. El paciente posteriormente muere de un fallo cardiorrespiratorio. El
examen de los pulmones en la autopsia lo más probable es divulgar cuál de las
siguientes alteraciones patológicas? (E) la congestión vascular y macrófagos
cargados de hemosiderina- edema pulmonar

88. Una mujer de 92 años de edad, es llevada inconsciente a la sala de emergencias de un


hogar de ancianos. Su presión arterial es 70/30 mm Hg. Ella se encuentra febril (38 °
C/100.5 ° F) y taquipnea. Los estudios de laboratorio muestran un recuento de
leucocitos de 22.000 / l con los neutrófilos del 92%. Un análisis de orina revela
numerosos organismos Gram-negativos. ¿Cuál de las siguientes cuentas en busca de
signos más probables de este paciente y los síntomas? (E) Shock séptico

89. Una mujer embarazada de 21 años de edad, sufre desprendimiento de la placenta a las
37 semanas de gestación y se desarrolla sangrado vaginal severo que es difícil de
controlar. Cinco meses más tarde, el paciente se presenta con profundo letargo,
palidez, debilidad muscular, falta de lactancia y amenorrea. ¿Cuál de los siguientes
que mejor explica la patogénesis de la hipófisis es una insuficiencia en este paciente?
(C) Infarto-hipófisis

90. Un hombre de 62 años de edad, con antecedentes de hipertensión se apresuró a la sala


de emergencia con graves "dolor desgarrador" de la parte anterior del tórax. Su
presión arterial es 80/50 mm Hg. El examen físico muestra palidez, diaforesis, y un
soplo de regurgitación aórtica. Los estudios de laboratorio y electrocardiograma no
muestran evidencia de infarto agudo de miocardio. Cuatro horas más tarde, el paciente
entra en paro cardíaco. Un ECG revela disociación electromecánica. ¿Cuál de los
siguientes que mejor explica la patogénesis de taponamiento cardíaco en este
paciente? (D) Hemorragia-aneurisma

91. Una mujer de 58 años de edad, se pone al servicio de urgencias 4 horas después de
vomitar sangre y experimentar heces sanguinolentas. La paciente fue diagnosticada
con cirrosis alcohólica hace 2 años. La endoscopia revela grandes varices esofágicas,
uno de los cuales está sangrando activamente. ¿Cuál de los siguientes que mejor
explica la patogenia de la dilatación de las venas esofágicas en este paciente? (C)
Aumento de la presión hidrostática intravascular-varices esofágicas

92. Un jubilado de 69 años de edad, se pone al servicio de urgencias debido a la


aparición repentina de dolor en el pecho del lado izquierdo, que se ve agravada en la
inspiración. El examen físico revela la disnea y hemoptisis. Su temperatura es de 38 °
C (101 ° F), pulso 110 por minuto, frecuencia respiratoria 35 por minuto y la presión
arterial 158/100 mm Hg. Una pared lateral del tórax roce está presente en la
auscultación. La pierna izquierda es marcadamente edematosa con un signo de
Homans positivo ". Un pecho de rayos-X revela un derrame pleural izquierdo. ¿Cuál
es la causa más probable de la afección pulmonar de este paciente? (E)
Tromboembolismo

93. A, trabajador de la construcción de 22 años de edad, cae de 30 pies y se fractura


varios huesos, incluyendo sus ejes femorales. Seis horas más tarde, el paciente
presenta dificultad para respirar y cianosis. ¿Cuál de los siguientes trastornos
hemodinámicos que mejor explica la patogénesis del shock en este paciente? (C) las
grasas embolia- fractura de los huesos largos

94. Una mujer de 20 años de edad, se presenta a la sala de urgencias quejándose de haber
tenido un fuerte dolor de cabeza durante 4 horas. El examen físico revela numerosas
pequeñas manchas rojas en las extremidades y el cuello rígido. Su temperatura es de
38,7 ° C (103 ° F). La punción lumbar vuelve purulenta fluido, con los neutrófilos
segmentados y organismos gram-negativos se asemejan a los meningococos. Unas
horas más tarde, el paciente entra en shock y se convierte en estado de coma. Una
lesión grave del endotelio en este paciente está mediado principalmente por cuál de
las siguientes proteínas?(E) factor de necrosis tumoral!- shock séptico , meningitis

95. Un hombre de 69 años de edad, es llevado a la sala de urgencias quejándose de Visual


dificultad y debilidad. En el examen físico, el paciente es afásica con una hemiplejía
del lado derecho. Hemorragias de la retina se ven bilateral. Se sospecha que un
tromboembolismo corría hacia la izquierda y la arteria cerebral media más pequeña
embolia viajó a las arterias de la retina. ¿Cuál de los siguientes sitios anatómicos es la
fuente más probable de estos émbolos en este paciente? (C) Corazón

96. El cuerpo de un hombre sin hogar de 28 años de edad, es llevado a la oficina del
médico forense. El examen histológico de los pulmones bajo la luz polarizada se
muestra. ¿Cuál de las siguientes es la causa más probable de la birrefringencia
observada en esta lesión pulmonar? E) uso de drogas intravenosas

97. Una mujer de 25 años de edad, entrega un bebé sano a las 39 semanas de gestación.
Seis horas más tarde, la madre desarrolla una intensa falta de aire y aparece cianosis.
A pesar de la reanimación, muere 2 horas más tarde. Una sección de pulmón en la
autopsia se muestra en la imagen. Estos hallazgos patológicos están asociados con
cuál de los siguientes mecanismos de la enfermedad? Embolia de líquido amniótico

98. Si el paciente se describe en la pregunta 11 haber sobrevivido a un episodio agudo de


cianosis y shock, ella habría estado en riesgo de desarrollar cuál de las siguientes
complicaciones potencialmente mortales? (B) la coagulación intravascular
diseminada-embolia de líquido amniótico

99. Un hombre de 68 años de edad con cardiopatía isquémica y una historia de quejas de
fumar de mayor dificultad para respirar. En el examen físico, el paciente tiene las
piernas hinchadas, un agrandamiento del hígado y fluido en los espacios pleurales
(burbujas estertores se escuchan en auscultation). ¿Cuál de los siguientes trastornos
hemodinámicos, explica la patogénesis de la hepatomegalia en este paciente? (B) la
congestión pasiva crónica

100. El paciente descrito en la pregunta 13 sufre un ataque masivo al corazón y expira.


El examen microscópico del hígado en la autopsia lo más probable es que se revelan
de la siguiente histopatológico cambios? (E) sinusoides dilatados con sangre-la
congestión pasiva crónica

101. Una mujer de 33 años de edad, presenta heces negras. Los estudios de laboratorio
muestran una anemia hipocrómica microcítica. La endoscopia GI superior revela una
úlcera duodenal. ¿Cuál de las siguientes opciones describe mejor la materia fecal en
este paciente con enfermedad de úlcera péptica? (D) Melena

102. Un hombre de 53 años de edad, se encuentra hospitalizado luego de lesionarse el


cuello en un accidente automovilístico. Él se coloca en tracción cervical. Una semana
después, el paciente desarrolla una hinchazón dolorosa y eritema de la pantorrilla
izquierda. Doppler revela la trombosis venosa profunda. ¿Cuál de las siguientes es la
causa más probable para el desarrollo de trombosis en este paciente? (E) por
estasis

103. Un hombre de 23 años con hemofilia es poco en silla de ruedas. ¿Cuál de las
siguientes cuentas mejores de este desarrollo? (A) Hemartrosis

104. A 50 años de edad, surge de una casa en llamas con quemaduras de tercer grado
con más de 70% de su cuerpo. El paciente expira 24 horas más tarde. ¿Cuál de las
siguientes es la causa más probable de la muerte? (C) shock
hipovolémico-hemorragia

105. Una mujer de 23 años de edad, se queja de la reciente aparición de coloración


amarillenta de la piel y el aumento de circunferencia abdominal. El examen físico
muestra ictericia y ascitis. La ecografía del abdomen muestra la trombosis de las
venas hepáticas. Una biopsia hepática revela severa dilatación sinusoidal, dentro de
las regiones centrolobulillar. Este hallazgo patológico es causado por ¿Cuál de los
siguientes trastornos hemodinámicos? (E) hiperemia pasiva

106. Una mujer de 42 años de edad, se somete a un lavado de cara. Dos días más tarde,
se presenta para la atención de seguimiento con el confl hemorragias uent azuladas en
la piel alrededor de los ojos ("ojos negros"). ¿Cuál de las siguientes opciones describe
mejor este patrón de hemorragia superficialmente la piel social? Equimosis-
hemorragia superficial en la piel

107. Una mujer de 19 años de edad, se queja de hinchazón de los párpados, el


abdomen y los tobillos. A la hora de acostarse, hay depresiones en las piernas en lugar
del elástico en sus calcetines. Una radiografía de tórax mostró derrame pleural
bilateral. Electroforesis de proteínas en orina muestra proteinuria 4 +. Una biopsia
realizada a una el riñón establece el diagnóstico de síndrome nefrótico cambio
mínimo. El edema de partes blandas en este paciente es muy probablemente causado
por el cual de los siguientes mecanismos de la enfermedad? (C) Disminución de la
presión oncótica intravascular

108. Un alcohólico de 50 años de edad, es llevado al hospital con hemorragia por


várices esofágicas y expira. En la autopsia, abdomen protuberante del paciente se
encontró que contenía un gran volumen de seroso uid fl. ¿Cuál es el término
apropiado para describir este líquido? (A) Ascitis

23 -Una niña de 1 año de edad, es llevada a la sala de emergencia por sus padres que
informan de que ha tenido fiebre y diarrea durante 3 días. Su temperatura es de 38 ° C (101 °
F). El CSC muestra un recuento normal de glóbulos blancos y aumento de hematocrito (48 g /
dl). ¿Cuál de las siguientes es la causa más probable del aumento de hematocrito en este
paciente? (B) Deshidratación

24 - Un hombre de 40 años de edad, con antecedentes de endocarditis bacteriana da cuenta de


numerosas hemorragias puntiformes alrededor de la órbita de sus ojos (que se muestra en la
imagen, vea las flechas). ¿Cuál es el término apropiado para describir esta forma de
hemorragia superficial? Petequias- endocarditis, hemorragias en la piel

25 - Un hombre de 67 años de edad se presenta con un dolor repentino pierna izquierda,


ausencia de pulsos, y la extremidad fría. Su historia clínica es significativa para la
enfermedad de la arteria coronaria y un aneurisma aórtico pequeño. ¿Cuál de los siguientes es
más probable responsable del Desarrollo de un miembro de frío en este paciente? (B)
tromboembolismo arterial-

26-Una mujer de 78 años de edad, muere en su sueño. Una mancha azul de Prusia de los
pulmones en la autopsia se muestra en la imagen. ¿Cuál de las siguientes es la causa más
probable de estos hallazgos histopatológicos? (B) La insuficiencia cardíaca congestiva
27-Un hombre de 60 años de edad, quien se recupera de una cirugía para corregir un
aneurisma abdominal que repentinamente presenta dolor torácico agudo y muere. Un
tromboembolismo en la bifurcación de las arterias pulmonares derecha e izquierda se observa
en la autopsia (que se muestra en la imagen). ¿Cuál de las siguientes es la causa más probable
de la embolia pulmonar de este paciente? La trombosis venosa profunda- trombolismo
pulmonar

28-Un hombre de 20 años de edad, es llevado a la sala de emergencia tras romperse el bazo
en un accidente de motocicleta. Su presión arterial al ingreso es 80/60 mm Hg. El análisis de
gases en sangre arterial demuestra acidosis metabólica. Este paciente es el sufrimiento más
probable es que a partir de cuál de las siguientes condiciones? (D), shock hipovolémico

29 - Un hombre de 72 años de edad, ha muerto a la llegada después de desmayarse en su


casa. Infartos corticales renales se observó en la autopsia. Una sección a través de la arteria
arqueada se muestra. ¿Cuál de las siguientes es la fuente más probable de la oclusión de esta
arteria atheroembolism? (A) la aorta abdominal

30 - Una mujer de 72 años de edad, se queja de falta de aire al hacer esfuerzos. La autora
afirma que también se convierte en falta de aire durante la noche a menos que ella utilice tres
almohadas (ortopnea). El examen físico revela la obesidad leve, edema bilateral de la pierna
picaduras, un agrandamiento del hígado y el bazo, y fi ne crujido suena en la inspiración
(estertores). Una radiografía de tórax muestra cardiomegalia. ¿Cuál es la causa más probable
de ortopnea en este paciente? (E) El edema pulmonar- fallo congestivo del corazón

31- Un hombre de 63 años de edad, sufre un derrame cerebral masivo y expira. En la


autopsia, el patólogo puso un trombo laminado adherido a la pared del ventrículo izquierdo
(como se muestra en la imagen). ¿Cuál de las siguientes es la causa más probable de este
hallazgo la autopsia? infarto al miocardio - ventrículo izquierdo

32 - El examen histológico del corazón en el paciente descrito en la pregunta 31 muestra el


crecimiento extensivo de fibroblastos y el depósito de colágeno en el trombo mural. ¿Cuál de
los siguientes términos describe el resultado de la trombosis? (C) Organización

33 - Una mujer de 50 años de edad se presenta con la fatiga y dificultad para respirar. El
examen físico muestra evidencia de edema pulmonar, aumento del tamaño de la aurícula
izquierda, y la calcificación de la válvula mitral. Una tomografía computarizada muestra una
gran masa de obstrucción en la aurícula izquierda. Antes de la cirugía a corazón abierto se
puede realizar para extirpar el tumor, el paciente sufre un derrame cerebral y expira. ¿Cuál de
los siguientes trastornos hemodinámicos que mejor explica la patogenia del ictus en este
paciente? (A) La embolia arterial- máxima cardiaco
34 - Una mujer de 50 años de edad, aparece en su oficina. Fue sometida a una mastectomía
radical y disección de ganglios axilares del cáncer de mama hace un año. Ahora se da cuenta
de que el brazo se hincha por el final del día. ¿Cuál es el nombre apropiado para esta
acumulación de fluido? (B) El linfedema

35 - Un hombre de 68 años de edad, se desarrolla un dolor repentino y severo en el pecho


subesternal. Estudios de laboratorio y ECG confirman un infarto agudo de miocardio. A pesar
del tratamiento vigoroso, el paciente no puede mantener su presión arterial y termina 24 horas
después. Una sección transversal del ventrículo izquierdo se examina en la autopsia (que se
muestra en la imagen). Las flechas apuntan a una superficie suave, de color amarillo de la
necrosis. ¿Cuál de las siguientes es la causa más probable de la muerte? (A) El shock
cardiogénico- falla en el miocardio

36 - Una mujer de 80 años de edad con antecedentes de hipertensión arterial se apresuró a la


sala de urgencias quejándose de dolor en el pecho de una duración de 1 hora. El examen
físico revela edema bilateral de la pierna picaduras, hepatoesplenomegalia y estertores en las
bases de ambos pulmones. El paciente es aprensivo y sudoración. El paciente pierde el
conocimiento y muere de una arritmia cardíaca. El examen microscópico de los pulmones en
la autopsia se muestra. ¿Cuál de los siguientes procesos hemodinámicos explica mejor esta
patología ending fi? (C) Aumento de la presión hidrostática intravascular- edema
pulmonar- izquierdo ventrículo

37 - Un niño de 9 meses de edad, es llevado a la sala de urgencias con una historia de 3 horas
de intenso dolor abdominal y diarrea con sangre. El examen físico revela sensibilidad en el
abdomen sin ascitis. El niño muere 24 horas después, y la torsión (vólvulo) del intestino
delgado que se descubre en la autopsia. El intestino delgado se ve dilatado y hemorrágico
(que se muestra en la imagen). ¿Cuál de las siguientes opciones describe mejor estos
hallazgos patológicos? infarto-(vólvulo)- colitis isquémica

38 - La autopsia de una mujer de 70 años de edad, revela un infarto subendocárdico


circunferencial del ventrículo izquierdo. Este tipo de infarto se asocia más comúnmente con
cuál de las siguientes? (B) shock hipotensivo

39 - Una mujer de 76 años de edad, se pone al servicio de urgencias debido a la repentina


aparición de dos episodios de hemoptisis y dolor en el lado izquierdo del pecho, que se ve
agravada en la inspiración. Su temperatura es de 38 ° C (101 ° F), pulso 110 por minuto,
frecuencia respiratoria 35 por minuto y la presión arterial 158/100 mm Hg. El paciente es
admitido, pero sufre un derrame cerebral masivo y expira 48 horas más tarde. La autopsia
revela un infarto pulmonar en segmentos superiores del lóbulo inferior (que se muestra en la
imagen). ¿Cuál de los siguientes que mejor explica el color de infarto pulmonar de este
paciente? (C) La hemorragia de las arterias bronquiales
40 - Una mujer de 22 años da a luz a un bebé a las 29 semanas de gestación. Poco después del
nacimiento, el recién nacido se queda sin aliento. El recién nacido es colocado en un
respirador artificial, pero se muere de insuficiencia respiratoria. El cerebro en la autopsia se
muestra. ¿Cuál de los siguientes mecanismos de la enfermedad que mejor explica esta
complicación del síndrome de dificultad respiratoria (SDR) del recién nacido? (A)lesión
anóxica

41- Masculino de 75 años fumador con hipertensión automedicada con homeopatía. El


último mes tuvo un cuadro clínico que inició con disnea y tos no productiva. El cuadro
evolucionó con marcado edema las regiones medias e inferior de los pulmones y a la
auscultación identificó estertores. Las últimas semanas presentó marcada cianosis, dificultad
respiratoria y falleció. En la autopsia se encontró lesión pulmonar que corresponde a la
imagen. Relacionado con la imagen microscópica cuál de los siguientes es el diagnóstico?
Daño alveolar difuso

42. Femenina de 6 meses con historia de dos días de tos severa, sibilancias y vías
respiratorias. El examen físico muestra rinitis, cianosis leve y fiebre. cuál de los siguientes
es la etiología más probable de infección pulmonar de esta niña? Virus sincitial respiratorio

43. Una niña de 6 meses como agrandamiento gradual del abdomen por hepatomegalia
masiva. Los estudios muestran niveles séricos normales de albúmina, bilirrubina, y enzimas
hepáticas. La biopsia hepática revela hepatocitos agrandados con inclusiones PAS positivas
con depósitos anormales de glucógeno (amilopectina). También se encuentran estos
depósitos en el tejido muscular. Cuál es el diagnóstico más probable? Enfermedad de
Andersen

Masculino de un 1 presenta retraso en el desarrollo motor, debilidad muscular progresiva,


ceguera y fallece. el cerebro el autopsia muestra inflamación y las neuronas contienen
numerosos lisosomas llenos de lípidos. Cuál de los siguientes es el diagnóstico más
probable? Enfermedad de tay-sachs
Paciente de 29 años con antecedentes familiares de neurofibromatosis, quién comenzó a notar
la presencia de tumores blandos subcutáneos no dolorosos diseminados en el tronco y en las
cuatro extremidades asociado a dolor de radicular y parestesias, dificultad para la marcha
atrofia muscular. Al examen físico estos tumores son suaves bien delimitados del mismo
color de la piel y de tamaños variables. Además presenta nódulos pigmentados en el Iris
(nódulos de Lisch) se diagnostica con una enfermedad de origen genético. cuál de los
siguientes es el diagnóstico más probable? Sindrome de Von Recklinghausen

Su médico le recomienda a un hombre hace como 3 años que se recuperaron infarto de


miocardio que tomé una aspirina todos los días para reducir la posibilidad de desarrollar un
segundo infarto de miocardio. La teoría detrás de este consejo es que la Aspirina disminuye
la formación de trombos dentro de las arterias coronarias. La formación de qué sustancia
inhibe la Aspirina? Tromboxano

Masculino de 19 años presenta palidez generalizada de la piel por lo que acude al doctor se
realiza hemograma que reportarlo óvulos blancos en 98,000, hemoglobina en 9.3 y HTC en
23% y conteo de plaquetas en 102000, este paciente tiene antecedentes de neurofibromatosis
tipo 1. A juzgar por su diagnóstico principal. ¿Cuál de las siguientes patologías es más
probable que este paciente? Leucemia mielocitica

Masculino de 9 meses de edad es llevado al pediatra para presentar distensión abdominal. Al


examen físico presenta esplenomegalia y hepatomegalia. Este cuadro clínico fue empeorando
con el paso de los meses ahora presenta además flexión hacia atrás del cuello, estrabismo y
dificultad para abrir la boca y tragar. paciente fallece a los 2 años y medio de vida. Se realiza
estudio genético y están unos días con una enfermedad autosómica recesiva. A juzgar por el
comportamiento que tuvo el paciente. ¿Cuál de los siguientes cree usted que es el
diagnóstico más probable? Enfermedad de gaucher tipo 2

Masculino de 10 años con historia de haber iniciado a la edad de 13 años con trastornos
visuales y que luego presentó hipertensión arterial severa. El examen físico era de elevada
estatura (2.03 m) con extremidades largas y pechugas excavatum. cuál de los siguientes es la
causa molecular de la enfermedad de este paciente? Mutación en el gen que codifica la
fibrina 1, en el brazo largo del cromosoma 15.

Lactante de 5 meses de nacido es llevado a consulta de neurocirugía por presentar gran lesión
quística en la región lumbosacra que hace 2 días se había fisurado y dejaba en evidencia la
presencia de Raíces nerviosas mezclada con el líquido cefalorraquídeo. De las siguientes,
cuáles error de morfogénesis cree usted que responsable de esta condición? Disrupción

Masculino de 48 años refiere debilidad muscular generalizada. diagnosticado hace 3 años


con cisticercosis en varios músculo del cuerpo. se realiza el electrocardiograma que revela
lesiones cisticas en el deltoide y los aductores de ambas piernas. ¿Cuál de los siguientes
hallazgos microscópicos guarda relación con las manifestaciones clínicas de esta
enfermedad? Atrofia muscular
Adolescente de 12 años acude a la consulta de pediatría presentar una fiebre de 38 - 39 C,
acompañado de astenia, odinofagia y una adenopatía en la región cervical derecha.
Presentando 7 días de secreción y obstrucción nasal acompañado de dolor en hemiabdomen
superior. Al examen físico estaba febril 39 C. Orofaringe enrojecida, amígdala derecha
aumenta de tamaño con el exudado blanquecino espeso. adenopatía cervical posterior de 2
cm de diámetro. Ecografía abdominal reportó una esplenomegalia ligera. En el extendido
periféricos evidencia múltiples linfocitos atípicos. Se sospecha de mononucleosis infecciosa.
¿Cuáles de los siguientes linfocitos cree usted que es el que más está pareciendo de manera
típica en sangre periférica? Linfocitos t activados

Un niño de 9 años desarrolla repentinamente dolor testicular severo. Lo llevaron a la sala de


emergencia donde lo evaluaron y lo llevaron inmediatamente a cirugía. Allí se encuentra que
su testículo izquierdo es notablemente hemorrágico debido a la torsión testicular. cuál de los
siguientes mecanismos está involucrado principalmente en la producción de este tipo de
infarto testicular? Oclusión venosa

Niña de 8 años llevada a consulta con historia de leve traumatismo le produce heridas
importantes, hemorragias cutáneas y hematomas. al examen físico llama la atención la
presencia de numerosas cicatrices irregulares, cubiertas de piel fina con aspecto de papel de
seda. Hay hiperelasticidad de la piel con pliegues que al soltarlos la piel recupera
inmediatamente su forma normal sin dejar arrugas. La articulación se puede extender y rodar
en exceso sin provocar dolor. En la cara se aprecia ligero epicantus y nariz fina. La paciente
refiere intenso dolor muscular y de articulaciones frecuentes. Cuál de los siguientes es el
diagnóstico más probable? Síndrome de Ehlers Danlos

Es una forma seria del síndrome de Ehlers Danlos Que se produce por defecto en el colágeno
tipo 3 por mutación del COL3A1, herencia autosómica dominante y cursa con
complicaciones vasculares: Ehlers Danlos tipo 4

Femenina 25 años gesta partos, llega la emergencia obstetricia en pleno labor de parto se
realiza parto eutócico Con recién nacido masculino de apgar 9-10. Una hora más tarde la
madre de torna cianótica con dificultad para respirar, Seguida de coma que no responde a
maniobras de resucitación. En la autopsia a nivel de los pulmones evidencia. A qué
corresponde ? Embolia líquido amniótico

Una niña recién nacida está siendo evaluada por problemas respiratorios y se encuentra que
tiene paladar hendido y micrognatia. Su mandíbula muy pequeña causó el desplazamiento
posterior de su lengua, que respiración y la deglución. En base a estos hallazgos físicos, se
realiza el diagnóstico de la secuencia de Pierre Robin. Cuál de las siguientes es la mejor
definición del término clínico secuencia? La combinación de un defecto primario junto
con sus cambios estructurales secundarios
Femenina de 7 años de vida presenta Fiebre de 7 días de evolución, vómito, malestar
general y lesiones vesiculares diseminadas en toda la piel. es diagnosticada con
varicell-zoster. De los siguientes, cuál es la característica citopática de este virus?
Inclusiones intranucleares

Masculino 70 años con historia de disnea, ortopnea, hepatomegalia, distensión de las


venas del cuello de 3 meses de evolución, acompañado de edema periférico. Fallece por
insuficiencia respiratoria secundaria a un edema pulmonar. se realiza autopsia que muestra
superficie del corte del hígado mostrada en la imagen. ¿Cuál de los siguientes trastornos es la
causa más probable de estos hallazgos? Insuficiencia cardíaca derecha

Un jugador ofensivo de 19 años para un importante equipo universitario de fútbol fractura su


fémur derecho durante el primer juego de la temporada. Es ingresado en el hospital y en los
próximos días desarrollará problemas respiratorios progresivos. A pesar de una extensa
intervención médica muere tres días después. En el momento de la autopsia, se observa
material oil red positiva en los pequeños vasos sanguíneos de los pulmones y el cerebro. cuál
de los siguientes es el diagnóstico más probable? Émbolos grasos
1. Masculino de 70 años de edad, que vivió 60 años en una aldea de nativos africanos
(sub- saharan), acude al médico por cuadro clínico de ascitis, trastorno la función
hepática, elevación de las enzimas hepáticas, sin anticuerpos para virus de hepatitis C
ni B niega consumo de alcohol. Se le diagnostica con carcinoma hepatocelular el
paciente tuvo exposición a un agente carcinogénico en su región natal cuál casino.
¿Cual carcinogénico está vinculado al desarrollo de este cáncer?
a. Nikel
b. Nitrosamina
c. Hidrocarburos aromáticos
d. Aflatoxina B1
2. Pertenece al tipo II, de hipersensibilidad tipo citotóxico
a. anemia hemolítica autoinmune
b. Enfermedad de graves
c. reacción de arthus
d. ReAcción de anafilaxia
3. La infección por genotipos de alto riesgo de virus papiloma humano está relacionada con el
carcinoma de células escamosas por degradación de genes supresores tumorales ¿Cuál de las
siguientes mecanismos explicaría mejor esta asociación?
a. HHV-8
b. OncoProteinas Virales E6-E7
c. Translocación en c-Myc
d. Antigenos nucleares
4. Masculino de 7 años de edad presenta lesión de color café oscuro 8 mm de diámetro, de
bordes irregulares, dolorosa en la pared del tórax, se realiza biopsia que reporta un melanoma
y antecedentes de recibir tratamiento para un carcinoma de células escamosas hace 1 ano.
Este paciente tiene la condición de origen hereditario que su doctor le explica que es un factor
predisponente para desarrollar este tipo de cáncer de las siguientes: ¿ Cuál cree usted que es
la enfermedad genética que más probablemente tenga este pac iente?
a. Anemia de fanconi
b. xeroderma pigmentoso
c. esclerosis tuberosa
d. sindrome de peutz-jeghers
5. Masc. de 28 años con múltiples lesiones osteolíticas en cráneo y columna vertebral
Calcificaciones en los en los túbulos renales. Es diagnosticado con mieloma múltiple. El calcio
sérico es ( 8,5 a 10,2). la anormalidad metabólica descrita aquí es probablemente debido a la
sustancia?
a. péptido de hormona paratiroidea
b. hormona antidiurética
c. factor de activación osteoclastos
d. hormona adrenocorticotropa
6. Una mujer de 54 años de edad que ha sido diagnosticada con cáncer de mama en etapa
temprana se somete a una cirugía para una tumorectomía para extirpar un pequeño tumor
decretado por monografía. El informe de patología confirma la etapa temprana del cáncer
,comentarios adicionales sobre el hecho de que hay desmoplasia significativa en ell tejido
circundante. el término desmoplasia se refiere a:
a. una acumulación y irregular de vasos sanguíneos
b. maduración y Disposición espacial de las células
c. proliferación de tejido conectivo fibroso no neoplasico
d. afectación metastásica de tejido circundante
7. Paciente masculino de 52 años, fumador de tabaco y cigarrillo, presenta carcinoma de
células pequeñas en pulmón izquierdo con metástasis de diversos órganos. Cuál de las
siguientes cree usted que fue la vía inicial más común para metástasis de cáncer?
a. Por invasión local
b. Hematógena
c. Linfática
d. a través de los bronquios
8. Masculino de 34 años de edad, fumador, consulta por presentar ascitis de 3 meses de
evolución y dificultad respiratoria. al examen físico, presenta taquipnea Hepatomegalia. luego
de múltiples estudios se le realiza biopsia peritoneal en un tumor seroso con abundante
democracia y proliferación vascular haciéndose diagnóstico De mesotelioma bien diferenciado.
¿Cuál de los siguientes marcadores tumorales nos ayudaría a establecer el diagnóstico de
esta neoplasia?
a. HMB-45
b. CA19-9
c. Cytokeratins CK5/6
d. a-Fetoprotein (AFP)
9. Femenina de 16 años de edad de raza negra Quien presenta eritema facial Como alas de
mariposas e inflamación de articulaciones es referida consulta de reumatología por diagnóstico
de lupus eritematoso sistémico. Diga a cuál tipo de hipersensibilidad pertenece:
a. Tipo I
b. Tipo IV
c. Tip o III
d. Tipo II
10. Femenina de 36 años que refiere haber sido diagnosticada con una inmunodeficiencia
congénita a la edad de 8 años pero que siempre ha estado asintomática. ahora presenta
procesos alérgicos y recurrentes y sindrome de malabsorcion desde hace 4 meses. ¿Cuál de
las siguientes es el diagnóstico más probable?
a. Síndrome de diGeorge
b. Agammaglobulinemia ligada al x de bruton
c. deficiencia selectiva de IgA
d. sindrome de hiper-IGM
11. Masculino de 78 años con una historia de fumador de 6000 paquetes de cigarrillos por año,
se presenta a la emergencia con dificultad para respirar y hemoptisis. Radiografía de tórax
muestra una gran masa centrada en el interior del campo pulmonar izquierdo. El calcio sérico
es de 13, 0 mg/dl ( normal 8,5 a 10,2). la anormalidad metabólica descrita aquí es
probablemente debido a la elaboración de qué sustancia?
a. Antigeno carcinoembrionario
b. hormona relacionada con la paratiroides
c. hormona antidiurética
d. eritropoyetina
12. Masculino de 44 años diagnosticado con glomerulonefritis membranosa por la cual recibió
trasplante renal izquierdo como tratamiento definitivo, elegante fue uno de sus tres hijos. acude
a consulta de nefrología por presentar cuadro clínico de fatiga y aumento de urea y creatinina
luego de 3 semanas del trasplante. la biopsia renal mostró infiltrado inflamatorio expensas de
linfocitos, macrófagos y necrosis cortical. de acuerdo de sus hallazgos clínicos y patológicos,
diga cuál tipo de rechazo de trasplante se refiere el caso.
a. Hiperagudo
b. Agudo
c. Inmediato
d. Cronico
13.Masculino de 64 años de edad tiene una larga historia de cáncer de prostata metastasico en
el momento del diagnóstico. Durante los últimos dos meses, ha tenido pérdida significativa de
peso, pérdida de apetito y pérdida de energía. su actual condición de caquexia se puede
atribuir A cuál de los siguientes?
a. Interleucina-2
b. factor de crecimiento derivado de plaquetas
c. factor de necrosis tumoral-a
d. factor de crecimiento de fibroblastos
14.Femenino de 46 años de edad, diabética, la cual presenta como complicación una
enfermedad renal crónica por nefropatía membranosa recibiendo trasplante renal de su
esposo. dos meses tras el trasplante paciente presenta fiebre alta y oliguria de un día de
evolución, con Resultado de creatinina 5.0mg/Dl y urea en 200mg/Dl. Se realiza biopsia de
riñón trasplantado que muestra inflamación intestinal con infiltración de macrófagos y linfocitos,
edema, tubulitis intersticial, necrosis tubular y fibrinosa y daño vascular. ¿De Acuerdo a estos
hallazgos histológicos Qué tipo de rechazo a trasplante presenta Y por qué mecanismo se
produjo?
a. Rechazo crónico, humoral y celular
b. rechazo agudo, ambos mecanismos humoral y celular
c. precioso crónico, mecanismo humoral
d. rechazo hiperagudo, ambos mecanismos humoral y celular
15. Una mujer de 46 años que presenta una masa y irregular de 3 cm que no es móvil. Se
realiza una mastectomía y el patólogo en la sección de la mamá encuentra una masa de 3
x3,5cm que no tiene bordes discretos, pero parece infiltrarse en el estroma mamario
circundante. la masa es firme, blanca y tiene consistencia fibrosa. ¿ cuándo las siguientes
características se demuestran con mayor probabilidad por la aparición de esta masa?
a. Desmoplasia
b. Anaplasia
c. Metaplasia
d. displasia
16. es correcto sobre los oncogenes:
a. Genes celulares normales cuyos productos promueven la proliferación celular
b. su estudio no es relevante la oncología moderna
c. una proteína codificada por un oncogen que impulsa la proliferación celular a través de
uno de varios mecanismos
d. versiones mutadas que funcionan de manera autónoma, habiendo perdido la
dependencia de señales normales que promueven el crecimiento.
17. Fem. 80 años presenta pérdida de sangre en heces, Refiere que su familia hay historia de
pólipos. En la colonoscopia se observa masa tumoral en su colon ascendente, se diagnóstica
con adenocarcinoma. ¿ Cuál de las siguientes antioncogenes ( gen supresor tumoral) es más
probable que esté mutado?
a. APC
b. Rb
c. WT1
d. PTEN
18. Femenina de 43 años de edad Presenta una masa palpable de 4cms de diámetro en el
cuadrante externo a la mama izquierda, se realiza biopsia y reporta un carcinoma ductal
infiltrante todo lo que se decide mastectomía con disección de los ganglios linfáticos axilares. ¿
Cuál de las siguientes sería el mejor predictor de buen pronóstico en este paciente?
a. No Metástasis en los ganglios linfáticos
b. el tamaño del tumor menor de 5 centímetros
c. las células tumorales responden fuertemente a estímulos estrogénicos
d. no antecedentes del cáncer de mama y la familia
19. Masc. 65 Años con sangrado gastrointestinal. la colonoscopia reporta una lesión triploide a
nivel de Colón descendente, Cuál de las siguientes características histopatológicas definirá
mejor que sea maligna o benigna:
a. Revestimiento epitelial con atipia
b. estructuras glandulares irregulares
c. Mitosis
d. invasión al estroma

roja= no estoy 100 segura


amaricclo= correctas
Examen Parcial Inmunopatología y Neoplasia 20 pts.
 Pregunta 1
1- Masculino de 70 años de edad, que vivió 60 años en una aldea de
nativos africanos(sub-Saharan), acude al médico por cuadro clínico de
ascitis, trastornos de la función hepática, elevación de las enzimas
hepáticas, sin anticuerpos para virus de Hepatitis C ni B y niega
consumo de alcohol. Se le diagnóstica con carcinoma hepatocelular. Es
probable que el paciente tuvo exposición a un agente carcinogénico en
su región natal. ¿Cuál carcinogénico está vinculado al desarrollo de
este cáncer?
Respuestas seleccionadas:
D. Aflatoxina B1
Respuestas correctas:
D. Aflatoxina B1
 Pregunta 2
12.Fem. 80 años presenta pérdida de sangre en heces, refiere que en su
familia hay historia de pólipos. En la colonoscopia se observa masa
tumoral en su colon ascendente, se diagnóstica con adenocarcinoma.
¿Cuál de los siguientes antioncogenes (gen supresor tumoral) es más
probablemente que este mutado?
Respuestas seleccionadas:
A. APC

Respuestas correctas:
A. APC

 Pregunta 3
Femenina de 36 años que refiere haber sido diagnosticada con
una inmunodeficiencia congénita a la edad de 8 años pero que
siempre ha estado asintomática. Ahora presenta procesos
alérgicos recurrentes y síndrome de malabsorción desde hace 4
meses. ¿Cuál de las siguientes es el diagnóstico más probable?
Respuestas seleccionadas: C. Deficiencia selectiva de IgA
Respuestas correctas: C. Deficiencia selectiva de IgA
 Pregunta 4
Masculino de 78 años con una historia de fumar de 6000 paquetes de
cigarrillos por año, se presenta a la emergencia con dificultad para
respirar y hemoptisis. Radiografía de tórax demuestra una gran masa
centrada en el interior del campo pulmonar izquierdo. El calcio sérico
es de 13,0 mg / dl (normal 8,5 a 10,2). La anormalidad metabólica
descrita aquí es probablemente debido a la elaboración de qué
sustancia?
Respuestas seleccionadas:
A. Hormona relacionada con la paratiroides
Respuestas correctas:
A. Hormona relacionada con la paratiroides
 Pregunta 5
Masc.de 28 años con múltiples lesiones osteolíticas en cráneo y
columna vertebral e insuficiencia renal con calcificación en los túbulos
renales. Es diagnosticado con Mieloma Múltiple. El calcio sérico es de
13,0 mg / dl (normal 8,5 a 10,2). ¿La anormalidad metabólica descrita
aquí es probablemente debido a la elaboración de qué sustancia?
Respuestas seleccionadas:
D. Factor de activación osteoclastos
Respuestas correctas:
D. Factor de activación osteoclastos
 Pregunta 6
Es correcto sobre los oncogenes:
Respuestas C. versiones
mutadas que funcionan de manera
seleccionadas:
autónoma, habiendo perdido la dependencia de señales
normales que promueven el crecimiento.
Respuestas C. versiones
mutadas que funcionan de manera
correctas:
autónoma, habiendo perdido la dependencia de señales
normales que promueven el crecimiento.
 Pregunta 7
11. Femenina de 46 años de edad, diabética, la cual presenta
como complicación una enfermedad renal crónica por nefropatía
membranosa recibiendo trasplante renal de su esposo. Dos meses
tras el trasplante paciente presenta fiebre alta y oliguria de un día
de evolución, con resultado de creatinina en 5.0 mg/Dl Y Urea en
200 mg/Dl. Se realiza biopsia del riñón trasplantado que muestra
inflamación intersticial con infiltración de macrófagos y linfocitos,
edema, tubulitis intersticial, necrosis tubular y fibrinosa y daño
vascular. ¿De acuerdo a estos hallazgos histológicos que tipo de
rechazo a trasplante presenta y por qué mecanismo se produjo?
Respuestas C. Rechazo crónico, humoral y celular
seleccionadas:

Respuestas correctas: B. Rechazo agudo, ambos mecanismos humoral


y celular
 Pregunta 8
Masculino de 64 años de edad tiene una larga historia de cáncer de
próstata metastásico en el momento del diagnóstico. Durante los
últimos 2 meses, ha tenido pérdida significativa de peso, pérdida de
apetito y pérdida de energía. Su actual condición de caquexia se puede
atribuir a cuál de los siguientes?
Respuestas seleccionadas:
C. Factor de necrosis tumoral-α
Respuestas correctas:
C. Factor de necrosis tumoral-α
 Pregunta 9
Masculino de 34 años de edad, fumador, consultó por presentar
ascitis de 3 meses de evolución y dificultad respiratoria. Al
examen físico, presenta taquipnea y hepatomegalia. Luego de
múltiples estudios se le realiza biopsia peritoneal que un tumor
seroso con abundante desmoplasia y proliferación vascular
haciéndose diagnóstico de mesotelioma bien diferenciado. ¿Cuál
de los siguientes marcadores tumorales nos ayudaría a establecer
diagnóstico de esta neoplasia?
Respuestas seleccionadas: C. Cytokeratins CK5/6
Respuestas correctas: C. Cytokeratins CK5/6
 Pregunta 10
18. Paciente masculino de 52 años, fumador de tabaco y cigarrillo,
presenta carcinoma de células pequeñas en pulmón izquierdo con
metástasis a diversos órganos. ¿Cuál de las siguientes cree usted
que fue la vía inicial más común para metástasis de este cáncer?
Respuestas seleccionadas: D. Linfática

Respuestas correctas: D. Linfática


 Pregunta 11
17. Femenina de 43 años de edad presenta una masa palpable de
4 cms de diámetro en el cuadrante externo de la mama izquierda,
se realiza biopsia que reporta un carcinoma ductal infiltrante por lo
que se decide mastectomía con disección de los ganglios
linfáticos axilares. ¿Cuál de las siguientes sería el mejor predictor
de buen pronóstico en esta paciente?
Respuestas seleccionadas: D. No metástasis en los ganglios linfático
Respuestas correctas: D. No metástasis en los ganglios linfático
 Pregunta 12
Masc. 65 años con sangrado gastrointestinal. La colonoscopia reporta
una lesión poliploide a nivel de colon descendente, cuál de las
siguientes características histopatológicas definirá mejor que sea
maligna o benigna:
Respuestas seleccionadas:
D. Invasión al estroma
Respuestas correctas:
D. Invasión al estroma
 Pregunta 13
13. Masculino de 13 años, HIV positivo, presenta dificultad respiratoria,
hemoptisis, tos y fiebre alta, su doctor piensa que puede estar cursando
con una tuberculosis por lo que realiza una prueba cutánea de la
tuberculina que resulta positiva, con un área firme de color rojo oscuro en
el antebrazo, aparece 60 horas después de la inyección del derivado
proteico purificado (PPD). ¿Este hallazgo es más probable que sea una
consecuencia de cuál de los siguientes tipos de reacción de
hipersensibilidad?
Respuestas seleccionadas: D. Hipersensibilidad de tipo IV
Respuestas correctas: D. Hipersensibilidad de tipo IV
 Pregunta 14
15. La infección por genotipos de alto riesgo del virus del papiloma
humano está relacionada con el carcinoma de células escamosas
por degradación de genes supresores tumorales. ¿Cuál de los
siguientes mecanismos explicaría mejor esta asociación?
Respuestas seleccionadas: B. Oncoproteínas virales E6-E7
Respuestas correctas: B. Oncoproteínas virales E6-E7
 Pregunta 15
Masculino de 44 años diagnosticado con glomerulonefritis
membranosa por la cual recibió trasplante renal izquierdo como
tratamiento definitivo, el donante fue uno de sus 3 hijos. Acude a
consulta de nefrología por presentar cuadro clínico de fatiga y
aumento de urea y creatinina luego de 3 semanas del trasplante. La
biopsia renal mostró infiltrado inflamatorio a expensas de
linfocitos, macrófagos y necrosis cortical. De acuerdo a estos
hallazgos clínicos y patológicos, diga cual tipo de rechazo de
trasplante se refiere el caso.
Respuestas seleccionadas:
B. Agudo

Respuestas correctas:
B. Agudo

 Pregunta 16
Femenina de 16 años de edad de raza negra quien presenta eritema
facial como alas de mariposas e inflamación de articulaciones es
referida a consulta de reumatología por diagnóstico de Lupus
Eritematoso Sistémico. Diga al cual tipo de hipersensibilidad
pertenece:
Respuestas seleccionadas:
A. Tipo III
Respuestas correctas:
A. Tipo III
 Pregunta 17
20. Masculino de 7 años de edad, presenta lesión de color café
oscuro, 8 mm de diámetro, de bordes irregulares, dolorosa en la
piel del tórax, se realiza biopsia que reporta un melanoma y
antecedente de recibir tratamiento para un carcinoma de células escamosas
hace 1 año. Este paciente tiene una condición de origen hereditario
que su doctor le explica es un factor predisponente para
desarrollar este tipo de cáncer. De las siguientes, ¿cuál cree usted
que es la enfermedad genética que más probablemente tenga
este paciente?
Respuestas seleccionadas: C. Xeroderma pigmentoso
Respuestas correctas: C. Xeroderma pigmentoso
 Pregunta 18
Pertenece al tipo II de hipersensibilidad, tipo citotóxico:
Respuestas seleccionadas:
A. Anemia hemolítica autoinmune
Respuestas correctas:
A. Anemia hemolítica autoinmune
 Pregunta 19
19. Una mujer de 46 años presenta una masa irregular de 3 cm
que no es móvil. Se realiza una mastectomía y el patólogo en la
sección de la mama encuentra una masa de 3 x 3,5 cm que no
tiene bordes discretos, pero parece infiltrarse en el estroma
mamario circundante. La masa es firme, blanca y tiene una
consistencia fibrosa. ¿Cuál de las siguientes características se
demuestra con mayor probabilidad por la apariencia de esta
masa?
Respuestas seleccionadas: D. Metaplasia

Respuestas correctas: B. Desmoplasia

 Pregunta 20
Una mujer de 54 años de edad que ha sido diagnosticada con cáncer de
mama en etapa temprana se somete a una cirugía para una
tumorectomía para extirpar un pequeño tumor detectado por
mamografía. El informe de patología confirma la etapa temprana del
cáncer y comentarios adicionales sobre el hecho de que hay
desmoplasia significativa en el tejido circundante. El término
desmoplasia se refiere a:
Respuestas B. proliferación de tejido conectivo fibroso no
seleccionadas:
neoplásico.
Respuestas correctas:
B. proliferación de tejido conectivo fibroso no
neoplásico.
enfermedades hemodinamicas

1 - Un hombre de 60 años de edad con un historial de múltiples infartos de miocardio es


hospitalizado por falta de aire. El examen físico revela marcada distensión yugular,
hepatomegalia, ascitis y edema con fóvea. Una radiografía de tórax muestra cardiomegalia. El
paciente posteriormente muere de un fallo cardiorrespiratorio. El examen de los pulmones en la
autopsia lo más probable es divulgar cuál de las siguientes alteraciones patológicas?

(E) la congestión vascular y macrófagos cargados de hemosiderina- edema pulmonar

2 - Una mujer de 92 años de edad, es llevado inconsciente a la sala de emergencias de un


hogar de ancianos. Su presión arterial es 70/30 mm Hg. Ella se encuentra febril (38 ° C/100.5 °
F) y taquipnea. Los estudios de laboratorio muestran un recuento de leucocitos de 22.000 / l
con los neutrófilos del 92%. Un análisis de orina revela numerosos organismos Gram-
negativos. ¿Cuál de las siguientes cuentas en busca de signos más probables de este paciente
y los síntomas?

(E) Shock séptico

3 -Una mujer embarazada de 21 años de edad, sufre desprendimiento de la placenta a las 37


semanas de gestación y se desarrolla sangrado vaginal severo que es di fícil de controlar.
Cinco meses más tarde, el paciente se presenta con profundo letargo, palidez, debilidad
muscular, falta de lactancia y amenorrea. ¿Cuál de los siguientes que mejor explica la
patogénesis de la hipófisis insufi ciencia en este paciente? (C) Infarto-hipófisis

4-Un hombre de 62 años de edad, con antecedentes de hipertensión se apresuró a la sala de


emergencia con graves "dolor desgarrador" de la parte anterior del tórax. Su presión arterial es
80/50 mm Hg. El examen físico muestra palidez, diaforesis, y un soplo de regurgitación aórtica.
Los estudios de laboratorio y electrocardiograma no muestran evidencia de infarto agudo de
miocardio. Cuatro horas más tarde, el paciente entra en paro cardíaco. Un ECG revela
disociación electromecánica. ¿Cuál de los siguientes que mejor explica la patogénesis de
taponamiento cardíaco en este paciente?

(D) Hemorragia-aneurisma

5 - Una mujer de 58 años de edad, se pone al servicio de urgencias 4 horas después de


vomitar sangre y experimentar heces sanguinolentas. La paciente fue diagnosticada con
cirrosis alcohólica hace 2 años. La endoscopia revela grandes varices esofágicas, uno de los
cuales está sangrando activamente. ¿Cuál de los siguientes que mejor explica la patogenia de
la dilatación de las venas esofágicas en este paciente?
(C) Aumento de la presión hidrostática intravascular-varices esofágicas

6 - Un jubilado de 69 años de edad, se pone al servicio de urgencias debido a la aparición


repentina de dolor en el pecho del lado izquierdo, que se ve agravada en la inspiración. El
examen físico revela la disnea y hemoptisis. Su temperatura es de 38 ° C (101 ° F), pulso 110
por minuto, frecuencia respiratoria 35 por minuto y la presión arterial 158/100 mm Hg. Una
pared lateral del tórax roce está presente en la auscultación. La pierna izquierda es
marcadamente edematoso con un signo de Homans positivo ". Un pecho De rayos-X revela un
derrame pleural izquierdo. ¿Cuál es la causa más probable de la afección pulmonar de este
paciente?

(E) Tromboembolismo

7-A, trabajador de la construcción de 22 años de edad, cae de 30 pies y se fractura varios


huesos, incluyendo sus ejes femoral. Seis horas más tarde, el paciente presenta dificultad para
respirar y cianosis. ¿Cuál de los siguientes trastornos hemodinámicos que mejor explica la
patogénesis del shock en este paciente? (C) las grasas embolia- fractura de los huesos
largos

8 - Una mujer de 20 años de edad, presenta a la sala de urgencias quejándose de haber tenido
un fuerte dolor de cabeza durante 4 horas. El examen físico revela numerosas pequeñas
manchas rojas en las extremidades y el cuello rígido. Su temperatura es de 38,7 ° C (103 ° F).
La punción lumbar vuelve purulenta fl uido, con los neutrófilos segmentados y organismos
gram-negativos se asemejan a los meningococos. Unas horas más tarde, el paciente entra en
shock y se convierte en estado de coma. Una lesión grave del endotelio en este paciente está
mediado principalmente por cuál de las siguientes proteínas?

(E) factor de necrosis tumoral!- chock séptico , meningitis

9-Un hombre de 69 años de edad, es llevado a la sala de emergencias quejándose de Visual


difi cultad y debilidad. En el examen físico, el paciente es afásica con una hemiplejía del lado
derecho. Hemorragias de la retina se ven bilateral. Se sospecha que un tromboembolismo
corría hacia la izquierda y la arteria cerebral media más pequeña embolia viajó a las arterias de
la retina. ¿Cuál de los siguientes sitios anatómicos es la fuente más probable de estos émbolos
en este paciente?

(C) Corazón
10 - El cuerpo de un hombre sin hogar de 28 años de edad, es llevado a la ofi cina del médico
forense. El examen histológico de los pulmones bajo la luz polarizada se muestra. ¿Cuál de las
siguientes es la causa más probable de la birrefringencia observada en esta lesión pulmonar?

E) uso de drogas intravenosas

11- Una mujer de 25 años de edad, entrega un bebé sano a las 39 semanas de gestación. Seis
horas más tarde, la madre desarrolla una intensa falta de aire y aparece cianosis. A pesar de la
reanimación, se muere 2 horas más tarde. Una sección de pulmón en la autopsia se muestra
en la imagen. Estos hallazgos patológicos están asociados con cuál de los siguientes
mecanismos de la enfermedad? embolia de líquido amniótico-

12 - Si el paciente se describe en la pregunta 11 había sobrevivido a un episodio agudo de


cianosis y shock, ella habría estado en riesgo de desarrollar cuál de las siguientes
complicaciones potencialmente mortales? (B) la coagulación intravascular diseminada-
embolia de líquido amniótico

13 - Un hombre de 68 años de edad con cardiopatía isquémica y una historia de quejas de


fumar de mayor dificultad para respirar. En el examen físico, el paciente tiene las piernas
hinchadas, un agrandamiento del hígado y fl uido en los espacios pleurales (burbujas estertores
se escuchan en oscultation). ¿Cuál de los siguientes trastornos hemodinámicos, explica la
patogénesis de la hepatomegalia en este paciente? (B) la congestión pasiva crónica

14 - El paciente descrito en la pregunta 13 sufre un ataque masivo al corazón y expira. El


examen microscópico del hígado en la autopsia lo más probable es que se revelan de la
siguiente histopatológico cambios? (E) sinusoides dilatados con sangre-la
congestión pasiva crónica

15 - Una mujer de 33 años de edad, presenta heces negras. Los estudios de laboratorio
muestran una anemia hipocrómica microcítica. La endoscopia GI superior revela una úlcera
duodenal. ¿Cuál de las siguientes opciones describe mejor la materia fecal en este paciente
con enfermedad de úlcera péptica?

(D) Melena
16 - Un hombre de 53 años de edad, se encuentra hospitalizado luego de lesionarse el cuello
en un accidente automovilístico. Él se coloca en tracción cervical. Una semana después, el
paciente desarrolla una hinchazón dolorosa y eritema de la pantorrilla izquierda. Doppler revela
la trombosis venosa profunda. ¿Cuál de las siguientes es la causa más probable para el
desarrollo de trombosis en este paciente? (E) por estasis

17-Un hombre de 23 años con hemofilia es poco en silla de ruedas. ¿Cuál de las siguientes
cuentas mejores de este desarrollo?

(A) Hemartrosis

18 - A 50 años de edad, re fi fi ghter surge de una casa en llamas con quemaduras de tercer
grado con más de 70% de su cuerpo. El paciente expira 24 horas más tarde. ¿Cuál de las
siguientes es la causa más probable de la muerte? (C) shock hipovolémico-hemorragia

19-Una mujer de 23 años de edad, se queja de la reciente aparición de coloración amarillenta


de la piel y el aumento de circunferencia abdominal. El examen físico muestra ictericia y ascitis.
La ecografía del abdomen muestra la trombosis de las venas hepáticas. Una biopsia hepática
revela severa dilatación sinusoidal, dentro de las regiones centrolobulillar. Este hallazgo
patológico fi es causada por ¿Cuál de los siguientes trastornos hemodinámicos?

(E) hiperemia pasiva

20 - Una mujer de 42 años de edad, se somete a un lavado de cara. Dos días más tarde, se
presenta para la atención de seguimiento con el confl hemorragias uent azuladas en la piel
alrededor de los ojos ("ojos negros"). ¿Cuál de las siguientes opciones describe mejor este
patrón de hemorragia superficialmente la piel social?

Equimosis- hemorragia superficial en la piel

21 - Una mujer de 19 años de edad, se queja de hinchazón de los párpados, el abdomen y los
tobillos. En la hora de acostarse, hay depresiones en las piernas en el lugar del elástico en sus
calcetines. Una radiografía de tórax muestra derrame pleural bilateral. Electroforesis de
proteínas en orina muestra proteinuria 4 +. Una biopsia realizada a una el riñón establece el
diagnóstico de síndrome nefrótico cambio mínimo. Edema de partes blandas en este paciente
es muy probablemente causado por el cual de los siguientes mecanismos de la enfermedad?
(C) Disminución de la presión oncótica intravascular

22-Un alcohólico de 50 años de edad, es llevado al hospital con hemorragia por várices
esofágicas y expira. En la autopsia, abdomen protuberante del paciente se encontró que
contenía un gran volumen de seroso uid fl. ¿Cuál es el término apropiado para describir este
líquido?

(A) Ascitis

23 -Una niña de 1 año de edad, es llevado a la sala de emergencia por sus padres que
informan de que ha tenido fiebre y diarrea durante 3 días. Su temperatura es de 38 ° C (101 °
F). El CSC muestra un recuento normal de glóbulos blancos y aumento de hematocrito (48 g /
dl). ¿Cuál de las siguientes es la causa más probable del aumento de hematocrito en este
paciente? (B) Deshidratación

24 - Un hombre de 40 años de edad, con antecedentes de endocarditis bacteriana da cuenta


de numerosas hemorragias puntiformes alrededor de la órbita de sus ojos (que se muestra en
la imagen, vea las flechas). ¿Cuál es el término apropiado para describir esta forma de
hemorragia superficial?

Petequias- endocarditis, hemorragias en la piel

25 - Un hombre de 67 años de edad se presenta con un dolor repentino pierna izquierda,


ausencia de pulsos, y la extremidad fría. Su historia clínica es significativo para la
enfermedad de la arteria coronaria y un aneurisma aórtico pequeño. ¿Cuál de los siguientes es
más probable responsable del Desarrollo de un miembro de frío en este paciente? (B)
tromboembolismo arterial-

26-Una mujer de 78 años de edad, muere en su sueño. Una mancha azul de Prusia de los
pulmones en la autopsia se muestra en la imagen. ¿Cuál de las siguientes es la causa más
probable de estos hallazgos histopatológicos? (B) La insuficiencia cardíaca congestive

27-Un hombre de 60 años de edad, quien se recupera de una cirugía para corregir un
aneurisma abdominal que repentinamente presenta dolor torácico agudo y muere. Un
tromboembolismo en la bifurcación de las arterias pulmonares derecha e izquierda se observa
en la autopsia (que se muestra en la imagen). ¿Cuál de las siguientes es la causa más
probable de embolia pulmonar de este paciente?

La trombosis venosa profunda- trombolismo pulmonar

28-Un hombre de 20 años de edad, es llevado a la sala de emergencia tras romperse el bazo
en un accidente de motocicleta. Su presión arterial al ingreso es 80/60 mm Hg. El análisis de
gases en sangre arterial demuestra acidosis metabólica. Este paciente es el sufrimiento más
probable es que a partir de cuál de las siguientes condiciones?

(D), shock hipovolémico

29 - Un hombre de 72 años de edad, ha muerto a la llegada después de desmayarse en su


casa. Infartos corticales renales se observó en la autopsia. Una sección a través de la arteria
arqueada se muestra. ¿Cuál de las siguientes es la fuente más probable de la oclusión de esta
arteria atheroembolus?

(A) la aorta abdominal

30 - Una mujer de 72 años de edad, se queja de falta de aire al hacer esfuerzos. La autora
afirma que también se convierte en falta de aire durante la noche a menos que ella utiliza tres
almohadas (ortopnea). El examen físico revela la obesidad leve, edema bilateral de la pierna
picaduras, un agrandamiento del hígado y el bazo, y fi ne crujido suena en la inspiración
(estertores). Una radiografía de tórax muestra cardiomegalia. ¿Qué es la causa más probable
de ortopnea en este paciente?

(E) El edema pulmonar- fallo congestivo del corazón

31- Un hombre de 63 años de edad, sufre un derrame cerebral masivo y expira. En la autopsia,
el patólogo fi nds un trombo laminado adherido a la pared del ventrículo izquierdo (como se
muestra en la imagen). ¿Cuál de las siguientes es la causa más probable de este hallazgo la
autopsia?

infarto al miocardio - ventrículo izquierdo


32 - El examen histológico del corazón en el paciente descrito en la pregunta 31 muestra el
crecimiento extensivo de fi broblastos y el depósito de colágeno en el trombo mural. ¿Cuál de
los siguientes términos describe el resultado de la trombosis?

(C) Organización

33 - Una mujer de 50 años de edad se presenta con la fatiga y dificultad para respirar. El
examen físico muestra evidencia de edema pulmonar, aumento del tamaño de la aurícula
izquierda, y la calcificación de la válvula mitral. Una tomografía computarizada muestra una
gran masa de la obstrucción en la aurícula izquierda. Antes de la cirugía a corazón abierto se
puede realizar para extirpar el tumor, el paciente sufre un derrame cerebral y expira. ¿Cuál de
los siguientes trastornos hemodinámicos que mejor explica la patogenia del ictus en este
paciente?

(A) La embolia arterial- máxima cardiaco

34 - Una mujer de 50 años de edad, aparece en su ofi cina. Fue sometida a una mastectomía
radical y disección de ganglios axilares del cáncer de mama hace un año. Ahora se da cuenta
de que el brazo se hincha por el final del día. ¿Cuál es el nombre apropiado para esta
acumulación de fl uido?

(B) El linfedema

35 - Un hombre de 68 años de edad, se desarrolla un dolor repentino y severo en el pecho


subesternal. Estudios de laboratorio y ECG confi rmar un infarto agudo de miocardio. A pesar
del tratamiento vigoroso, el paciente no puede mantener su presión arterial y termina 24 horas
después. Una sección transversal del ventrículo izquierdo se examina en la autopsia (que se
muestra en la imagen). Las flechas apuntan a una superficie suave, de color amarillo de la
necrosis. ¿Cuál de las siguientes es la causa más probable de la muerte?

(A) El shock cardiogénico- falla en el miocardio

36 - Una mujer de 80 años de edad con antecedentes de hipertensión arterial se apresuró a la


sala de urgencias quejándose de dolor en el pecho de una duración de 1 hora. El examen físico
revela edema bilateral de la pierna picaduras, hepatoesplenomegalia y estertores en las bases
de ambos pulmones. El paciente es aprensivo y sudoración. El paciente pierde el conocimiento
y muere de una arritmia cardíaca. El examen microscópico de los pulmones en la autopsia se
muestra. ¿Cuál de los siguientes procesos hemodinámicos explica mejor esta patología nding
fi?

(C) Aumento de la presión hidrostática intravascular- edema pulmonar- izquierdo ventrículo

37 - Un niño de 9 meses de edad, es llevado a la sala de urgencias con una historia de 3 horas
de intenso dolor abdominal y diarrea con sangre. El examen físico revela sensibilidad en el
abdomen sin ascitis. El niño muere 24 horas después, y la torsión (vólvulo) del intestino
delgado que se descubre en la autopsia. El intestino delgado se ve dilatado y hemorrágico (que
se muestra en la imagen). ¿Cuál de las siguientes opciones describe mejor estos hallazgos
patológicos?

infarto-(vólvulo)- colitis isquemica

38 - La autopsia de una mujer de 70 años de edad, revela un infarto subendocárdico


circunferencial del ventrículo izquierdo. Este tipo de infarto se asocia más comúnmente con
cuál de las siguientes?

(B) shock hipotensivo

39 - Una mujer de 76 años de edad, se pone al servicio de urgencias debido a la repentina


aparición de dos episodios de hemoptisis y dolor en el lado izquierdo del pecho, que se ve
agravada en la inspiración. Su temperatura es de 38 ° C (101 ° F), pulso 110 por minuto,
frecuencia respiratoria 35 por minuto y la presión arterial 158/100 mm Hg. El paciente es
admitido, pero sufre un derrame cerebral masivo y expira 48 horas más tarde. La autopsia
revela un infarto pulmonar en segmentos superiores del lóbulo inferior (que se muestra en la
imagen). ¿Cuál de los siguientes que mejor explica el color de infarto pulmonar de este
paciente?

(C) La hemorragia de las arterias bronquiales

40 - Una mujer de 22 años luz a un bebé a las 29 semanas de gestación. Poco después del
nacimiento, el recién nacido se queda sin aliento. El recién nacido es colocado en un respirador
artificial, pero se muere de insufi ciencia respiratoria. El cerebro en la autopsia se muestra.
¿Cuál de las siguientes mecanismos de la enfermedad que mejor explica esta complicación del
síndrome de dificultad respiratoria (SDR) del recién nacido?
(A) lesión anóxica
REPASO EXAMEN FINAL PATOLOGIA

1- Examen de laboratorio, colesterol serico de 920ml/dl, la mama del niño y el abuelo tambien
tenían niveles elevados de colesterol. El paciente fue diagnosticado con: hipercolesterolemia
familiar ¿Cuál es el cromosoma afectado? El cromosoma 19. Por qué ocurre un infarto del
miocardio en una persona joven? Por la hipercolesterolemia familiar

2- Quien tiene afectado el cromosoma 22?

Neurofibromatosis tipo 2 tiene manchas color café

3- Quien tiene afectado el cromosoma 17? La neurofibromatosis tipo 1 NF1) se caracteriza por

(1)alteracion de neurofibromas, (2) las áreas de pigmentación oscura de la piel (manchas café
con leche), y (3) las lesiones pigmentadas de la iris (nódulos de Lisch). Es uno de los más
comunes trastornos autosómicos dominantes. Una de las principales complicaciones de la
neurofibromatosis tipo 1 (NF1), que ocurre en 3% a 5% de los pacientes, es la aparición de un
neurofibrosarcoma en un neurofibroma. NF1 también se asocia con una mayor incidencia de
otros tumores neurogénicos, como meningioma, glioma óptico, y feocromocitoma.

4- Un muchacho de 10 años, retrasado mental, pero que pueda llevar a cabo cosas diarias
como bañarse, vestirse, alimentarse. Al examen físico tiene braquicefalia, fisuras palpebrales
oblicuas. Síndrome de Down: trisomía 21. Si ese muchacho llega a los 40 años de que puede
sufrir? Alzheimer Qué condición maligna se da frecuentemente en este paciente? Leucemia
linfoide aguda ocurre frecuentemente en pacientes con síndrome de down. Por qué
ocurre un síndrome de Down? Por la no disyunción. Que tipo de anomalía cromosómica hay
en este paciente? trisomía del cromosoma 21

5- Un muchacho de 25 años, va a consulta por infertilidad, se le encuentra oligospermia, al


examen físico tiene ginecomastia bilateral, túbulos cicatrizados. ¿Cuál es el diagnóstico?
Klinefelter Que tipo de anomalía presenta el paciente? Trisomía sexual XXY

6- Un niño de 3 años desarrolla ataxia, convulsiones, risa inapropiada, un cariotipo de 46 XY

Diagnóstico? Angelman (es materno) ¿Cual es el tipo de desorden genético que tiene este
paciente? Expansión repetida de 3 nucleótidos (buscar cuales nucleótidos)

7- Un niño de 7 años tiene calambres musculares cuando juega, una biopsia de musculo
revela acumulación de glicógeno en el musculo esquelético, porque a las células les falta una
enzima. Cual es la enzima faltante en esta enfermedad? Fosforilaza muscular. ¿Que tipo de
enfermedad es esta? Glicogénesis. ¿Que glicogenosis afectando el musculo tiene?
Enfermedad de Mcardle tipo 5. ¿Si es una glicogenosis hepática de la tipo 1 como se llama?
Enfermedad de Von Gierke

8- Una mujer de 18 años da a luz a un feto muerto de 33 semanas de edad gestacional, el feto
tenia masas bilaterales en el cuello, tenia hidropesía masiva, anomalías cromosómicas fueron
encontradas consistente con un cromosoma sexual en anillo, y el feto fue diagnosticado como
síndrome de? Turner. Cual de las siguientes anomalías esta asociada con este síndrome?
Coartacion de la aorta Cuando un síndrome de Turner nace vivo se puede decir de un niño
que nacio y describir un Turner y decir que presentaba cianosis en las extremidades inferiores,
las cuales se sentían frias, y ahí se diagnostica: la coartación de la aorta.

9- Un niño de 2,300 gramos nació a las 37 semanas de edad gestacional de madre de 25 años
gesta 6, pario 2 aborto 3. Al nacimiento el niño tenia multiples anomalías congénitas, como
labio leporino bilateral, paladar hendido, microcefalia y polidactilia, orejas de implantación baja,
pene hiposplatico y criptorquidia. Diagnostico: Trisomia 13, síndrome de Patau

10- Una femenina prematura nace con dificultad respiratoria y cianosis, muriendo 14 horas
mas tarde. A la apariencia microscópica los pulmones se presentaban solidos, con alveolos
que parecían hendiduras y material de membrana hialina. Por que se produce este cuadro?
Por falta de surfactante. Con que se relaciona esta condición? inmadurez o prematuridad.
En que pacientes se pude ver con mas frecuencia? En prematuros

11- Un masculino de 1 año nacio con una mancha rojo vino en la cara, microscópicamente
estaba constituida por multiples canales vasculares pequeños con escaso espacio entre ellos
El diagnostico seria de? Hemangioma

12- un masculino de 13 años presento esplenomegalia y cambios esqueléticos debido a una


enfermedad de almacenamiento lisosomal no letal, la apariencia microscópica eran células
grandes de abundante citoplasma con apariencia de papel de seda arrugado. La enfermedad
es causada por la mutación en el gen que codifica la enzima? Glucocerebrosidasa
Diagnostico? Gaucher

13- Un hombre de 60 años que ha tenido una enfermedad cardiaca congestiva de larga data
se queja de dolor debajo del cuadrante costal derecho, el murió por un empeorameinto de su
falla cardiaca. En la autopsia el hígado estaba aumentado de tamaño y presentaba áreas
oscuras mezcladas con áreas claras en un patrón parecido a nuez moscada. Esta
característica es compatible con? Congestion pasiva del higado

14- Un muchacho de 15 años fue al medico a causa de fiebre, dolor de cabeza y constipación,
seguid por diarrea. Los antígenos febriles fueron positivos. Una biopsia de intestino es como
muestra la imagen. Diagnostico? Salmonelosis ¿Cual es el cambio que produce eso en los
tejidos? Hiperplasia linfoide y aumento de los linfocitos.

15- Una mujer de 63 años muere dos días después del inicio de una isquemia miocárdica
severa causada por una trombosis coronaria, en la autopsia los pulmones están muy pesados
y cuando se seccionan sale un liquido de la superficie. La apariencia microscopia son alveolos
con material dentro de estos. Diagnostico? Edema pulmonar ¿Que causa o explica este
edema? Aumento de la presión hidrostática vascular ¿Cual es la causa de ese edema, o
sea, que causa un edema pulmonar? fallo cardíaco izquierdo

16- Un hombre de 25 años va a la emergencia por fiebre, escalosfrios y brigor. Al examen se le


encuentra un soplo en el área de la valvula tricuspidea, su cara y sus extremidades están
enrojecidas, mientras que su presión es de 90/60. El examen de laboratorio revela conteo
elevado de leucocitos con polimorfonucleares. El cultivo muestra una bacteria gram negativa,
es tratado con antibiótico introvenosos pero su presión continua declinando al igual que
disminuye la producción de orina y el paciente muere 3 dias después. ¿Cual fue la causa de la
muerte del paciente? Shock séptico. ¿En que tipo de pacientes se ve generalmente esta
lesiones de endocarditis bacteriana en la valvula tricuspidea? pacientes con catéter,
ancianos y drogadictos

17- Se encontro un cadáver, en el área frontal tenia ese orificio (foto). Por las características de
ese orificio de que tipo es? Es un orificio de salida (porque es irregular y esta estrellado). Si
fuera de entrada estaría redondito y fuera herniado hacia adentro.

18- Una mujer de 35 años va a la emergencia con un sangrado y tiene candida. Que produce
la candida en los tejidos? Necrosis

19- Mujer tiene debilidad muscular y en el electroangiograma había una lesión quística en el
musculo deltoides? Cisticerdosis o triquinosis.

20- Un hombre es llevado a la emergencia porque se rompió el bazo en un accidente de


motocicleta, supresión sanguínea era de 80/60, y el análisis de los gases arteriales mostraron
una acidosis metabolica. ¿El paciente esta sufriendo de? shock hipovolemico.

Si les dicen: - Bacteria gram negativa: shock séptico - Ruptura del bazo, hemorragia interna:
shock hipovolémico - Un infarto: shock cardiogenico

21- Un paciente de 50 años alcohólico es llevado al hospital por un sangrado de varices


esofágicas y se muere. En la autopsia el paciente tenía un abdomen protruyente con gran
cantidad de líquido seroso dentro. Cual es el termino apropiado para este líquido
intraabdominal? Ascitis Es la acumulación de líquido en el espacio que existe entre el
revestimiento del abdomen y los órganos abdominales (la cavidad peritoneal).

22- Un hombre de 46 años con obesidad tipo 1 va al hospital, el dice que ha sido profesor por
mas de 25 años, el vivía una vida sedentaria, presenta moderada dificultad respiratoria,
taquicardia y presión alta. A nivel de las piernas se encuentran multiples varices superficiales y
profundas. Después de 24 horas de admitirlo en el hospital el paciente muere. ¿Cual es el
diagnostico mas probable para este paciente? Tromboembolismo pulmonar

23- Un adulto sano que esta corriendo en un maratón en el verano desarrolla piel seca y
caliente, deja de sudar, tiene acidosis láctica, hipocalcemia y rabdomiolisis.*( Es la
descomposición de las fibras musculares que ocasiona la liberación de los contenidos de
dichas fibras (mioglobina) en el torrente sanguíneo. La mioglobina es tóxica para el riñón y con
frecuencia causa daño renal.) ¿Cual es el diagnostico apropiado? Golpe de calor (insolación)

24- Un hombre de 60 años va al hospital con dificultad respiratoria y edema de la


extremidades inferiores. La radiografia de torax muestra congestion pulmonar y edema, tiene
cardiomegalia severa. El electrocardiograma revela afinamiento del septum ventricular con
fracción de eyección disminuida y dilatación cardiaca. ¿Que factores contribuyen a los
síntomas previamente descritos? Alcoholismo crónico o a un fallo ventricular izquierdo
crónico

25- Un hombre de 43 años va al medico por adormecimiento del segundo dedo. Un doppler
vascular revela una enfermedad oclusiva inflamatoria de la vasculatura de la pierna inferior.
Diagnostico? Estrectocos piogenes. Esta condición se asocia con: erisipela

26- Una mujer de 65 años con historia de uso crónico de tabaco, dolor toraxico y esputo
sanguinolento se le hace una radiografia que revela un carcinoma de células escamosas,. Cual
ambiente quimico esta relacionado con este diagnostico? Hidrocarburo aromatico

27- Un neonato muestra síntomas de retardo del crecimiento, ligero retardo mental y
dismorfologia Diagnostico? Alcoholismo fetal

28- Una mujer con obesidad tipo 1 de 27 años presenta dolor abdominal y diarrea. Un
sonograma abdominal muestra una masa homogénea encapsulada de 2cm que es compatible
con una lesión hepática benigna. Cual de los siguientes agentes esta relacionado con esta
lesion? Contraceptivos

29- Una mujer de 55 años presenta 5 meses de amenorrea, va a su ginecólogo quejándose de


calor, perdida del apetito sexual, e insomnio. El ginecólogo le prescribe tratamiento basado en
estrógeno y progestina. Diagnostico? menopausia ¿Que tipo de cáncer probablemente pueda
padecer esta paciente? Carcinoma de mama

30- Un masculino de 3 años fue al hospital por calambres abdominales, dificultad para hablar
e irritabilidad. El hemograma muestra anemia, pero no tiene rigidez de nuca. Una tomografía
de la cabeza muestra edema cerebral y aplanamiento de la cisura y compresión de los
ventrículos. Un team medico es enviado a la casa del paciente para evaluar su habitación en la
pared ellos encontraron materiales compatibles con fragmentos de pintura. Cual de las
siguientes sustancias esta realcionada con el caso? Plomo

31- Un hombre de 88 años es llevado inconciente a la emergencia desde un hogar de


ancianos, su presión es de 65/35, esta afebril y taquipneico, y el hemograma muestra un
conteo de globulos blancos de 22,000 con un 91% de neutrófilos. El urianalisis revela varios
organismos gram negavitos. Cual de los siguientes acontecen para los signos y síntomas de
este paciente? Shock séptico

32- Una mujer de 30 años se fracturo su pierna y unos pocos días después desarrollo un rash
petequial, ella tenia un problema respiratorio y signos neurológicos. Estudios de laboratorio
mostraban trombositopenia. Estos hallazgos son consistentes con? Embolismo graso

33- Mujer de 32 años se hizo un levantamiento facial, dos días después va al medico para
seguimiento y se encuentran áreas hemorrágicas azuladas confluentes alrededor de los ojos
(ojos negros) ¿Cual de los siguientes términos describe esa hemorragia superficial en la piel?
Equimosis

34- Una mujer de 33 años presenta heces oscuras, el hemograma revela una anemia
hipocromica. Un sangrado duodenal desde una ulcera péptica es diagnosticado. ¿Cual de los
siguientes describe mejor lo que le pasa a la paciente? Melena

35- Un hombre de 50 años va para un examen de rutina y tiene el hígado un poco agrandado.
Durante la visita el describe lo que su supervisor de trabajo le pone acerca de la exposición
crónica al PBC. ¿Este paciente tiene un riesgo elevado de sufrir que tumor en el hígado?
buscar

36- Una mujer de 45 años se le hace una recesión gástrica por un adenocarsinoma de
estomago. Ella ha estado bien por 5 años, pero ahora tiene anemia, fatiga, se queda de
cambios neurológicos leves. Los estudios de laboratorio de medula osea revelan pancitopenia
y otros hallazgos compatibles con una anemia megaloblastica. Ella probablemente esta
sufriendo de deficiencia de que vitamina? Vitamina B12

37- Una mujer de 75 años va al doctor porque tiene un dolor en el codo, ella tiene historia
clínica de artritis , recientemente había tenido un trauma en un brazo, en la artroscopia se ve
sangre en el codo. Es tratada sin ningun tipo de complicaciones. Que termino describe la
presencia de sangre en las articulaciones? Hemartrosis

38- Un niño de 3 años nacio a termino sin anomalía congénita, pero tenia solamente el 65%
del peso normal. Al examen muestra edema en las extremidades y un abdomen agrandado
con onda liquida positiva. La piel descamativa con área de despigmentación e
hiperpigmentacion Diagnostico? Kwashikor

39- Una muchacha de 18 años da a luz a por parto vaginal sin complicación a un bebe de
1,500 gramos, con una placenta que pesada 330 gramos. No se vio anomalía congénita. Cual
de las siguientes condiciones probablemente se corresponde con estos hallazgos? Retardo
del crecimiento intrauterino

40- Una muchacha estudiante de 18 años va a la clínica de la universidad porque tenia dolor
de cabeza severo, pero tenia rigidez de nuca, dice que otros dos estudiantes en su dormitorio
tenían síntomas similares. La temperatura era de 39.3 C, y al examen físico muestra rigidez
nucal, tiene purpura palpable en el tronco y en las extermidades inferiores. Una tinción de
gram negativo en un LCE tenia diplococos gram negativos. Cual de los siguientes tipos de
inflamación es mas probablemente asociado con las condiciones neurológicas de este
paciente? Inflamación necrotizante

Bacterias intracelulares: granulomatosas. Virus: mononuclear. Sifilis: plasmática. Bacterias


gram negativas o positivas extracelulares: Neutrofilos (te puede dar una supurativa o exudativa
o necrotizante).

41- Un hombre de 60 años se esta recuperando de una cirugía, esta en cama, súbitamente
tiene dolor agudo en el pecho y se muere. Un tromboembolo se encuentra en la bifulcacion de
la areria pulmonar derecha e izquierda en la autopsia. Cual de las siguientes cree usted que es
la causa de este tromboembolo pulmonar? Trombo venoso profundo

42- Una mujer negra obesa de 47 años que mide 1.62 metros va al medico por una fiebre alta,
ictericia y hemorragias conjuntivales petequiales. Ella es admitida en el hospital y se muere una
semana después. En la autopsia se encuentran hemorragias petequiales en varios órganos
además de necrosis aguda en el riñon. En el hígado y riñon se encontraron microorganismos
que teñian de negro con una tinción especial. Que tinción especial cree usted que se uso?
Warthin- Starry Diagnostico? Leptospirosis

1. Autopsia a nivel de pulmón, se ve un área como en cuña con la base hacia afuera y hacia
adentro el vértice ve hemorrágico con áreas rojizas. En que situación sucede esto?
Tromboembolismo de arteria pulmonar

2. Paciente con pancreatitis, inflamación en piernas, sensible y caliente. Con que se asocian
las lesiones de páncreas? Trombosis venosa.
3. Paciente con hipercolesterolemia familiar. Cual es la patogénesis genética?

4. Px con opacidad en la cornea. Que vitamina esta deficiente? Vit.A Retinol

5. Patrón de herencia de labio leporino? Multifactorial

6. Cual es el trastorno genético enel Fragile-X? Repetición de nucleótidos CGG.

7. Un niño con 70% de su peso, edematizado, abdomen grande, hipopigmentacion


descamación. Kwashiorkor

8. Px con trastorno de alimentación, diarreas crónicas y lesiones de la piel con necrosis,


además de cuadro de confusión mental? Deficiencia de tiamina

9. Px alcohólico, requiere vitamina, cual? Tiamina

a. Que es una laceración, abrasión, protrusión, herida incisa, que es un orificio de entrada/
salida

10. Pruebas en las que se detectaron mieldas en tejido hepático. Cual de las siguientes
condiciones es la causa inmediata de muerte? Infección

11. Mujer con tos persistente, fiebre durante la tarde y sudores nocturnos? Tuberculosis

12. Px con sida que no puede tragar, abre la boca y tiene una placa blanquecina – cándida
albicans

13. Mujer de 18 anos da a luz un feto femenino muerto a las tres semanas. El feto tiene una
anomalía cromosómica que consiste en un cromosoma sexual en anillo? Turner

14. Infante masculino de 37 semanas pesa 2,300g, tiene múltiples anomalías congénitas
incluyendo labio leporino, microcefalia, baja implantación de orejas, polidactilia, pene
hipoplastico, testículos no descendidos. Que tiene? Pateu cromosoma13

15. Que tipo de neoplasia maligna hay en Sindrome de Down? Leucemia

16. Que categoría se le da a los Klinefelter? Trisomía en los cromosomas sexuales.

17. Un niño de tres anos con ataxia, convulsiones y riéndose? Sindrome de Angelman

18. Mujer de 25 anos. Al examen físico: Esplenomegalia masiva, cambios esqueléticos debio a
una enfermedad por almacenamiento lisosomal no letal causada por una mutacion en el gen
que codifica la glucosidasa. De que enfermedad se trata? Gaucher

19. Glucogenosis
a. Muscular. Deficiencia de fosforilasa muscular? McArdle b. Hepatica

20. Que se ve histológicamente en un hígado en nuez moscada? Dilatación de la vena central


junto a una atrofia y necrosis de hepatocitos. Cual es la patogénesis de la hepatomegalia en
esta condición? congestión pasiva crónica.

21. Mujer se rompe pierna, después desarrolla rash petequial en el cuerpo, trombocitopenia?
embolismo graso

22. Mujer que se hizo cirugía facial, hemorragias azules alrededor de los ojos (dos ojos
negros)? Equimosis

23. Fisiopatología del edema pulmonar? Aumento de la presión hidrostática intravascular /


enshock séptico es aumento de la permeabilidad capilar

24. Como se le llama a la pupu negra? Melena

25. Que neoplasia hepática se relaciona con el PBC? Angiosarcoma de hígado.

26. Adulto sano corre, de momento tiene piel caliente, seca, hace acidosis láctica,
hipocalcemia, necrosis muscular. Cual es el fx apropiado? Golpe de Calor.

27. Hombre de 60 anos, dificultad respiratoria, edema de miembros inferiores, congestión y


edema en pulmones junto a cardiomegalia severa. EKG tiene afinamiento del septum,
dilatación cardiaca y poca eyección? Abuso de alcohol

28. Neonato con retardo de crecimiento, dismorfologia facial. La mama era adicta a varias
sustancias. El problema de este niño se debió a? Ingesta de alcohol, Alcoholismo fetal

29. Una muchacha de 15 anos con obesidad tipo I, tiene dolor abd y diarrea. Sonograma:
masa homogénea encapsulada en hígado compatible con adenoma hepático, con cual de los
sgtes agentes esta lesión esta relacionada? ---

30. Sra que asiste al medico quejandose de calor, perdida del apetito sexual? Menopausia; Se
le administra estrógeno y progestina, con que canceres están relacionados? Cáncer ovárico y
de mamas.

31. Nino de tres anos, con irritabilidad, anemia, tomo con edema cerebral, aplanamiento de
fisuras y aplanamiento de ventrículos, gris oscuro en las encias. Un grupo va a su casa a
evaluar su habitación, en la pared encontraron material compatible con? Plomo

32. Hombre alcohólico va al hospital por sangrado de varices esofágicas... abundante fluido
seroso? Ascitis

33. Una mujer embarazada de 25anos de edad, a las 16 semana se le hace una prueba que
sugiere una malformación del tubo neural en el feto. –defecto de 3 cms de tubo neural- .La
prueba de tamizaje que se le hizo mide los niveles séricos de? Alfafetoproteina; Que se le
administra? Acido fólico; Cuales son genes tienen que ver con esta malformación? Pax 3

34. Hombre de 25 anos, px alto, dedos largos, probablemente lleva una mutación en el gen
que codifica cual proteína? Fibrilina

35. Niño 12 meses con debilidad progresiva, perdida de visión, deterioro mental, disminución
de la actividad de la hexosaminidasa A – (Enf. De Tay S ). Cual de las siguientes opciones
explica mejor la patogénesis? Acumulación de sustrato no metabolizado

36. Recién nacido se queda sin aliento, retracción intercostal. –Síndrome de membrana hialina.
Se relaciona con: Inmadurez/Falta de surfactante.

37. Hombre 60 con historia de infarto al miocardio es hospitalizado por falta de aire. En el
examen físico muestra distención yugular marcada, edema con fovea, ascitis, cardiomegalia.
El paciente muere de fallo cardiorespiratorio. En la autopsia lo mas probable es encontrar:
congestion vascular y macrófagos con pigmentos en su interior. ¿Como se llama el
pigmento? Hemosiderina.

38. Recién nacido nace con disfunción motora severa que comprmete a los miembros
inferiores. Cuerpos vertebrales sin arcos posteriores en porción lumbar. Los defectos están
cubiertos por una fina membrana. El espacio debajo de la membrana contiene una masa de
meninges y medula espinal. Los padres quieren saber que sustancia reduce el riesgo de
malformaciones del tubo neural? Acido fólico.

39. Los padres de un bebe con labio leporino y paladar hendido visitan a un consejero en
genética pues quieren evitar que su futura descendencia sufra lo mismo.-Que suplemento
vitamínico hay que darle? B12; que enfermedad es esta? Síndrome de Turner por deficiencia
d B12.

40. Joven de 28 años va a emergencia y una hora después de experimentar una serie de
estudios se confirma que tiene infarto agudo del miocardio. Fallece 24 horas después por
arritmia. De que padece el px probablemente? Hipercolesterolemia Familiar.

41. 25 anos autismo y retraso mental, presenta aumento del perímetro craneal y macro-
orquidismo. Su tio materno se ve igualmente afectado.Cuales la causa mas probable de la
enfermedad en este px? La expansión de repetición de Trinucleotidos –Enf. frágil x

42. Vendiendo viveres, el caballo lo golpea y le fractura varios huesos, incluyendo la cabeza
femoral. Seis horas mas tarde el px presenta dificultad para respirar? Embolismo graso

43. Mujer de 25 anos de edad da a luz un bebe, seis horas mas tarde este desarrolla cianosis e
intensa falta de aire? Embolismo por líquido amniótico.

44. Hombre de 60 anos se recupera de cirugía pa corregir aneurisma abdominal;


repentinamente presenta dolor torácico agudo y muere. Un tromboembolismo en la bifurcación
de la izquierda de arterias pulmonares se observa. Cual es la causa de la embolia pulmonar?
trombosis venosa profunda.

45. Que tipo de shock ocurre en px con infarto agudo al miocardio? cardiogénico

46. 25 anos, fiebre alta, escalofríos, soplo en valvula triuspide, extremidades y cara sonrojadas,
presión 90/60, leucocitosis, crecimiento con bacterias gram negativas. Cual es la causa de
muerte? Shock séptico. -Cual de los sgtes es el factor de riesgo mas importante? Uso de IV
Drugs. Que tipo de daño se puede encontrar en los riñones? Necrosis tubular agua.

47. Mujer queja de debilidad muscular. Lesion quística en el musculo deltoides, se le extirpa y
se ve lo siquiente. Que es? Atrofia de las fibras musculares. DX: Tenia.

a. Ascaris se va a musculos o cerebro

48. Una mujer de 50 anos fue sometida a una mastectomía radical, se da cuenta que el brazo
al final del día esta sumamente hinchado. Como se llama a la acumulación de ese liquido en el
brazo? Linfedema

49. Una mujer de 65 anos con antecedentes de fumadora tiene historia de dolor de pecho y
esputo sanguinolento. Masa pulmonar con carcinoma de células escamosas. Cual de los
siguientes agentes químicos puede estar relacionado? Hidrocarburos aromáticos (benceno)

50. Fatiga y heces color oscuro. Anemia microcitica hipocromica. Deficiencia de: hierro

51. Se rescata niño que se perdió en bosque de canada en mes de febrero. Examen físico:
gangrena de dedos de las manos y los pies,cual delos siguientes mecanismos de lesión celular
juega el papel mas importante en la necrosis? –“Membrana de interrupción por cristales de
agua” – not sure

52. Hombre con depresión severa que se suicida dejando el carro encendido. Que sucede?
Desplazamiento de O2 en la Hemoglobina por CO

53. Dolor y entumecimiento del segundo dedo del pie. Enfermedad peculiar inflamatoria y
oclusiva de la vasculatura de la pierna. Enfermedad de Weger. Está asociada a: fumar

54. Electricista que sufre quemadura profunda en la mano es encontrado inconsciente. De que
murió? Arritmia cardiaca

55. Dermatitis, diarrea, demencia; solo come derivados del maiz: pelagra.

56. Enfermedad de Crohn y malabsorción de grasa severa--- fractura del cuello femoral---
deficiencia de vitamina D.

57. Dolor de cabeza intenso, rigidez nucal, fiebre, lesiones en todo el tronco y extremidades
inferiores. Tincion de gram negativo muestra diplococos. Meningitis meningococica--- tipo de
inflamación: exudativa.

58. Fiebre alta, ictericia, petequias, hemorragias, NECROSIS AGUDA EN RINONES, en el


hígado y los rinones se encontraron microrganismos que se tiñeron con Warthin Starry. Que
enf?. Enfermedad de weil.

59. Herpes Simple: daño citopático --- se fusionan las células y se forma una cavidad

Agentes Infecciosos: ¿Qué producen?

Salmonelosis produce hiperplasia linfoide.

Monilia--- Candidiasis--- produce necrosis

Malaria—hepatocitos muertos, hiperplasia de las células de Kupffer, que están fagocitando un


pigmento negro llamado hemozoina.

Cromoblastomicosis: hiperplasia epitelial, granulomas con microabscesos (centros necróticos).

Canalículo biliar hipertrófico: fasciola hepática--- produce fibrosis marcada.

Imágenes de Enfermedades Infecciosas

1. Cuerpos de Donovan (granuloma inguinal)

2. Neumonía con Neutrófilos

3. Granuloma

Taenia se va a músculo o a cerebro. Tomografía años después, denota calcificación. Escolex


del parasito (cabeza). En intestino produce proglotidos, que tienen sexo diferente.

Apendicitis--- Útero de áscaris y adentro se ven los huevos.

T. trichuria: superficie espinosa.

E. vermicularis: espículas laterales e intensa inflamación alrededor.

4. Formación de células gigantes en Herpes y destrucción de áreas

5. Piel con fluido dentro y formación de células gigantes de herpes

6. Citomegalovirus

7. Células gigantes de Warthin-Finkeldey en Sarampión


PREGUNTAS PATOLOGIA FINAL

1- Una mujer de 38 años va a emergencias por un embarazo no deseado. La señora dice que tiene
una pareja y que utiliza anticonceptivos orales, y que hace 3 meses que presentaba amenorrea y por
eso decidió realizarse una prueba de embarazo. Tiene un historial de tabaquismo de 2 cajas diarias. El
ginecólogo le dice que debe dejar el cigarrillo. A las 40 semanas nace el bebé de la paciente con un
bajo peso muy marcado y el pediatra le informa que el niño puede presentar alteraciones en el
desarrollo físico, cognitivo y emocional. ¿A qué se debe el bajo peso del neonato?

A. Los anticonceptivos orales

B. El tabaquismo

C. La edad de la madre

D. Herencia

E. El niño nació prematuro

2- Paciente masculino de 45 años es traído a emergencias debido a un dolor que no le permitía


moverse su esposa llamó al servicio de 911 al llegar a la clínica y hacerle estudios se nota piedras
pancreáticas y deficiencia de este mismo; la esposa también dice que el presenta delirios de
confabulación después de ser despedido de su trabajo hace 1 año. Cuál de los siguientes puede
provocar estos síntomas:

A. Alcoholismo crónico

B. Cocaína

C. Olermarcadores

D. Meningioma

3- Paciente masculino de 21 años es traído a emergencias por que se cree superman al examinar al
pacientes vez heridas en el antebrazo causada por un arma blanca al preguntarle al paciente el dice
que nada lo lastima porque es superman, el paciente también presenta taquicardia y hipertension.
Cuál de las siguientes sustancias provocaría esto:

A. Heroína

B. Cocaína

C. PCP o polvo de ángel

D. LSD

4-Paciente femenina de 24 años llega a emergencias por un hemoperitoneo al hacer la tomografía vez
una masa en el hígado se toma una biopsia y da positivo para un adenoma hepático benigno al
preguntarle si toma algún medicamento ella responde que sí que toma pastillas anticonceptivas. Con
qué tiempo está relacionada estas pastillas con el adenoma hepático benigno:

A. 2 años

B. 4 años

C. 10 años

D. 5 años

5- Paciente femenina de 17 años de edad , acude a consulta porque es sexualmente activa y no


quiere quedar embarazada por lo que se le recomiendan anticonceptivos orales. ¿Cuál de estas es
una característica de estos medicamentos?

A. Previenen enfermedades de transmisión sexual

B. El alcohol disminuye el efecto de las pastillas anticonceptivas

C. Aumenta el riesgo de colelitiasis si se utiliza mas de 4 años

D. Disminuye el riesgo de cancer de ovario y uterino

E. Aumenta el riesgo de cancer de ovario y uterino

6- Paciente de 43 años llega a emergencia con enrojecimiento, picazón e inflamación en el dorso de


sus manos, se observan lesiones en la piel. Trabaja hace 2 años en una fábrica de monedas y nos
indica que lleva más de 1 mes con esta lesión. Es diagnosticado con dermatitis debido a la exposición
a níquel. ¿Con cuál otra enfermedad puede estar en riesgo nuestro paciente?

A. Cáncer de rodilla

B. Úlcera gástrica

C. Cáncer de pulmón
D. Artritis gotosa

7- Hombre de 30 años se presenta a emergencias con una quemadura en el brazo izquierdo. El área
se encuentra inflamada y a las horas se forma una ampolla intacta. El paciente dice que se quemo
con el fuego de la estufa y que no duro mucho tiempo en contacto con la llama. ¿De qué grado es
esta lesión?

A. Primer Grado

B. Segundo Grado

C. Tercer Grado

D.Cuarto Grado

E. Grado cero

8- Paciente masculino de 30 años acude a consulta tras presentar una pérdida de peso repentina,
dificultad para respirar, y problemas para dormir. El paciente revela en la historia clínica que tiene
antecedentes de haber fumado tabaco durante 10 años y que ha sido diagnosticado con Diabetes
Miellitus desde temprana edad, tambien asegura no haber tenido éstos síntomas en República
Dominicana, donde residía desde su infancia hasta hace 5 meses cuando se mudó a Chile. Mientras
vivía en República Dominicana el paciente era donador de sangre, cada mes iba a la Cruz Roja y hacia
su aporte a la sociedad. En el examen examen físico la pared torácica se encuentra aumentada de
tamaño y el paciente niega sentir dolor a la palpación. Según los estudios de laboratorio el conteo de
glóbulos rojos es superior al rango normal. El paciente asegura que en su vida cotidiana no hace
ejercicio y tiene una dieta descontrolada desde hace 3 meses, lo que le llevó a visitar a su médico
hace un mes para tratar el aumento de peso que su estilo de vida estaba generando. Este cuadro se
debe a cuál de los siguientes:

A. Obesidad

B. Efectos adversos del cigarrillo

C. Trastorno relacionado con la altitud


D. Donaba sangre con frecuencia

9- Estás rotando por pediatría cuando una madre lleva a su hija de 6 años la cual se golpeó su
antebrazo mientras jugaba , el lugar de la zona del golpe se torno azul oscuro y varios días después
tiene una coloración amarilla. ¿Cómo explicas lo sucedido?

A. Existencia de sangre muy oxigenada fuera de los vasos sanguíneos

B. Inflamación cronica

C. La conversión de hemoglobinasobina a hemosiderina por los macrófagos

D. Presencia de bilirrubina

E. La desoxigenación de la sangre acumulada

10- Masculino afrolatinoamericano de 31 años con obesidad mórbida se presenta en consulta con
debilidad muscular, cansancio, dolor en la parte inferior de la espalda y cadera. Dice que es un gamer
de profesión, trabaja en su casa, no está casado y rara vez sale de día. ¿A qué se deben estos
síntomas?

A. Es el resultado de su obesidad mórbida

B. A la deficiencia de vitamina D

C. Pasa mucho tiempo en la silla sentado

D. A su mala alimentación

11- Paciente femenina de 32 años acude a consulta en su quinto mes de gestación, en la consulta el
médico le dice que el niño tiene aparentes malformaciones del cráneo. Al nacimiento el niño presenta
hidrocefalia, espina bífida, paladar endido y anomalías genitales. El médico realiza una historia familiar
y nadie en la familia tiene ninguna de estas condiciones. La madre dice no haber consumido ningún
tipo de medicamentos que pudieran provocar estos síntomas en su hijo, sin embargo antes de saber
que estaba embarazada consumía bebidas alcohólicas y fumaba en exceso, esto antes de cumplir los
dos meses de gestación. La paciente realizó un viaje a Europa durante su embarazo y entre los países
que visitó se encuentran Francia, Italia, Portugal y Ucrania, allí visitó entre otras cosas la planta
nuclear de Chernóbil. Los síntomas que presenta el neonato se deben a:

A. Consumo de cigarrillo por parte de la madre

B. Mutación de novo

C. Consumo de alcohol por parte de la madre

D. Exposición a un nivel de radiación superior al recomendado

12- Madre de un niño de 9 años te comenta en consulta que el mismo es “muy mañoso” al comer.
Cuando realizas el examen físico notas muchos moretones en su cuerpo , la madre te explica que
últimamente es muy común que choque o tropiece con objetos en su casa cuando cae la tarde-
noche. ¿Que crees cause estos síntomas ?

A. Kwashiorkor

B. Toxicidad por vitamina A

C. Keratomalacia por déficit de proteína

D. Keratomalacia por déficit de vitamina A

E. Déficit de vitamina B2

1. Joven de 25 años que se muda de a un pueblo en una alta montaña llega a la sala de urgencias con
disnea, la cual comenzó en el ejercicio y luego estaba presente durante el reposo. Al examen físico
mostró cianosis, taquicardia, taquipnea y febrícula y respiración silbante focal o difusa. El paciente
luego cae en coma y muere. En la autopsia, se aprecia edema pulmonar confluente intenso, exudados
proteináceos alveolares y formación de membranas hialinas ¿cuál de los siguientes es más probable a
afección del paciente?

A) Mal de montaña agudo

B) Edema pulmonar de las grandes alturas


C) Edema cerebral de las grandes Alturas

D) Edema sistémico por altitud elevada

2. A un operativo médico en una región remota de la República Dominicana se acercan varias madres
diciendo que sus hijos manifiestan hormigueo y adormecimiento en las extremidades, también se les
dificulta ponerse de pie debido a la hinchazón en los miembros inferiores, las madres también
manifiestan que hace ya un mes que sus hijos no reciben una buena alimentación. Los niños
probablemente sufren de:

A) Anorexia.

B) Beriberi.
C) Kwashiorkor.

D) Bulimia.

E) Marasmo

3. Paciente femenino de 30 años llega a sala de emergencias presentando pérdida de la visión en su


ojo derecho. Al realizar fundoscopia usted observa palidez en la retina. Exámenes subsecuentes
arrojan como resultado una obstrucción de la arteria central de la retina por un trombo ¿Esta
condición de tromboembolismo pudiera estar asociada a cuál de las siguientes?

A) aumento de niveles de calcio en sangre

B) uso de drogas intravenosas

C) Uso de testosterona sintética

D) Uso de anticonceptivos orales


4. Llega a su consulta una pareja joven buscando consejería acerca de métodos anticonceptivos.
Mientras le explica sobre los distintos métodos, la señorita le pregunta sobre la posibilidad de utilizar
pastillas anticonceptivas. Usted le asegura que es una opción adecuada, pero le advierte que el uso
prolongado de anticonceptivos:

A) podría ocasionar litiasis renal

B) puede generar malestar estomacal

C) podría ocasionar hirsutismo

D) aumenta el riesgo de trombosis venosa


5. Ana quien tiene 39 años de edad visita a su médico debido a que presenta un dolor de espalda
fuerte hace varios días atrás, en el transcurso de su historia clínica le comenta al doctor que presentó
una menopausia temprana. En vista de esto el médico le realiza varias pruebas de laboratorio, dentro
de ellas el examen de densidad mineral ósea (DMO), el cual da positivo para el diagnóstico de
osteoporosis ¿cuál de los siguiente puede provocar este diagnóstico?

a) Ejercicio extremo.

b) Consumo de alcohol

c) Dieta de vegetales

d) Consumo de tabaco/cigarrillo.
e) Constante estrés.

6. Paciente masculino de 37 años de edad se presenta a consulta en silla de rueda debido a que tiene
debilidad en los miembros inferiores y pérdida de sensibilidad en el mismo lugar, en vista de esto el
médico pre-diagnosticó una polineuropatía, pero para asegurar esto le indica hacerse unos exámenes
los cuales muestran deficiencia de tiamina y otras vitaminas del complejo B ¿Que probablemente le
provocó estas enfermeda?

a) Consumo de Isotretinoína.

b) Consumo de cigarrillo.

c) Consumo de litio.

d) Consumo excesivo de alcohol.

7. Una mujer de 44 años es llevada al hospital por su esposo debido a que se encuentra en un estado
de delirio y actúa de forma extraña, el lo describe como si su esposa estuviera perdida en el espacio y
tiempo, combinado con una paranoia y una ansiedad constante. Su esposa muere horas más tarde
debido a una arritmia cardiaca. Los médicos explican a los familiares que al parecer esta había estado
bajo efecto de una droga. ¿Cuál de las siguientes sustancias es la más probable que estuviera
consumiendo este paciente?
A) Heroína
B) Cocaína
C) Anfetamina

D) Alucinógenos

E) Disolventes orgánicos

8. Un trabajador industrial, manufacturero de zapatos es diagnosticado con envenenamiento por


benceno, uno de los químicos más ampliamente utilizado en procesos industriales. Como resultado
de este fenómeno, ¿qué tipo de anemia es más probable que desarrolle, si aún sigue expuesto a altas
concentraciones de benceno?

A) Anemia aplástica
B) Anemia ferropénica

C) Anemia hemolítica

D) Anemia perniciosa

E) Anemia drepanocítica

9. Jose Armando un joven masculino de 24 años se presenta en su consulta con dolor gastrointestinal
y una muñeca caída del lado derecho. Los análisis de sangre revelaron una anemia
microcíticohipocrómica. Más estudios también revelaron la presencia la formación de cuerpos de
inclusión en los núcleos de células del túbulo proximal (síndrome de Fanconi). ¿Toxicidad por cual
sustancia puede causar estos signos y síntomas?

A) Hierro

B) Cobre

C) Plomo
D) Mercurio

10.Manuela una mujer de 53 años es diagnosticada con cáncer de mama luego de varios días
sintiéndose una masa en su mama derecha. En las próximas semanas recibe radioterapia y un tiempo
después presenta dilatación en los vasos sanguíneos y fibrosis densa en la dermis. ¿Cuál de los
siguientes efectos adversos es más probable que se presente debido a la radioterapia?

A. Acabiosis

B. Dermatitis por radiación


C. Psoriasis

D. Cáncer en la piel

E. Esclerodermia

11. Paciente femenina de 29 años se presenta a la emergencia con un trastorno térmico acompañado
de un estado acelerado del metabolismo. Se observó que este grupo de síntomas se desarrollaron
luego de una anestesia que le fue administrada. que trastorno térmico puede estar presentando la
paciente?

A. Hipotermia

B. Quemadura

C. Golpe de calor

D. Sobredosis

E. Hipertermia maligna

12. Un joven de 20 años es parte de una pandilla callejera, llega a emergencia luego de una pelea. En
el examen físico se observa que tiene el ojo morado, tu le dices al paciente que volverá solo a su color
original. Quienes son los encargados de que este hematoma se regenere?

A. Oxígeno

B. Macrófagos y bilirrubina
C. Eosinófilos

D. Neutrófilos y bilirrubina

E. Linfocitos y macrófagos

1. Mujer de 25 años que ha estado deprimida en los últimos días, tras haber perdido su trabajo, es
encontrada en su garaje con todas las puertas y ventanas cerradas y su auto encendido , al examen
físico se aprecia que su piel tiene un color rosado y que ha fallecido por asfixia . ¿Que podría ser la
causa de muerte en esta mujer?

A) Intoxicación por monóxido de carbon


B) Asbestos

C) Infarto de miocardio

D) Alcohol

2. Hombre de 30 años de edad va de vacaciones con su esposa a conocer Cusco, Perú desde la
República Dominicana. Una hora después de aterrizar en Cusco intentó dormir durante la horas de
vuelo pero lo no logro, el día después de llegar a Cusco después de haber salido a conocer y hacer su
primera excursión en la ciudad, comienza a sentir mareos, nauseas, dolor de cabeza y debilidad. Los
niveles de saturación de oxígeno en sangre se encontraban en 87% (normal entre 95% y 100%).
¿Cual de estos sería el diagnóstico más probable en el paciente?

A) Infarto de miocardio

B) Mal de montaña agudo

C) Hipertermia maligna

3. Melanie es una niña de 3 años de edad tiene erosión de la superficie corneal áspera causada por
xeroftalmía. La queratomalacia produce cicatrización corneal con eventual ceguera después de 4
años. Este daño ocular se debe por una deficiencia de?

A) Vitamina C

B) Acido folico

C) Vitamina A

4. Femenina de 25 años de edad llega a consulta mostrando una actitud un poco paranoica ̈ y
bastante ansiosa, el examen físico y mental revela perdida del sentido del olfato, problemas para
tragar, y hemorragias nasales. ¿Cuál de las siguientes sustancias puede ser la que llevo a la paciente
a esta condición?

a) LCD

b) Heroína

c) Cocaína
d) Alcohol

5. Mujer de 33 años de edad da a luz, su marido al recibir al bebe se muestra bastante impactado al
verlo y pregunta al medico de turno por el aspecto del bebe (tono muscular bajo, ausencia del surco
nasolabial, ojos pequeños y rasgados y una cabeza por debajo del tamaño normal). El medico le
responde que su hijo tiene una de las siguientes condiciones. ¿Cuál es?

a) Consumo de cocaína durante el embarazo

b) Tratamiento hormonal a la madre durante el embarazo

c) Síndrome de alcohol fetal

d) Uso de misoprostol durante el embarazo

6. Paciente masculino de 23 años se presenta a la consulta de nutrición. El paciente mide 5 ́ 7 y pesa


210lb (obesidad), dice que desea bajar de peso, pero no puede dejar de comer ya que nunca esta
satisfecho. Tomando en cuenta el estilo de vida y patrones de la enfermedad, ¿Cuál de estas
hormonas que responsable de inhibir el hambre es responsable de la alta ingesta calórica de este
paciente?

A. Grelina

B. Leptina
C. Insulina

D. Glucagon

E. Somatostatina

7. Paciente masculino de 21 años llega a emergencias luego de ser agredido fisicamente por dos
asaltadores, dice que fue golpeado en la cara multiples veces. Presenta una acumulación de sangre
por la ruptura de los capilares y venulas que le confiere al lugar de la lesion una coloración negruzca
(ojo negro). Esta clase de lesion se denomina como:

A. Hematoma

B. Teratoma

C. Enteroma

D. Desgarre

E. Melanoma

8. Paciente femenina de 63 años de edad es llevada a emergencias luego de sufrir un infarto al


miocardio. Al examen fisico se encuentra salida y con falta de aliento, la paciente tiene sobrepeso. Su
presión arterial es de 150/90 y tiene niveles de colesterol de 240 mg/dl en sangre. A parte de los
factores de riesgo mencionados, ¿Qué otro factor de riesgo contribuiria a un infarto del miocardio en
pacientes?

A. Evaluaciones cardiológicas periódicas

B. Hacer una hora de ejercicio al día

C. Comer bajo en sal

D. Tabaquismo

E. Buena Hidratación

9. Una mujer de 65 años se presenta a su médico quejándose de una tos que no desaparece y un
dolor en el pecho que a menudo empeora con la respiración profunda, la tos o la risa. Después de
más exámenes, el paciente es diagnosticado con cáncer de pulmón. Después de varias semanas de
comenzar la radioterapia, presenta eritema leve y descamación seca alrededor de su cuello. ¿Cuál de
los siguientes efectos adversos es más probable que se presente debido a la radioterapia?

A) Calambres por calor

B) Dermatitis por radiación


C) Eczema

D) Hipertermia maligna

E) Daño por inhalación térmica

10. Un renovador de casa de 58 años se presenta a su médico quejándose de cólicos intensos,


náuseas, pérdida de peso y debilidad muscular. Análisis de sangre revela anemia hemolítica,
aminoaciduria, glucosuria e hiperfosfaturia (síndrome de Fanconi). ¿Toxicidad por cual sustancia
puede causar estos signos y síntomas?

A) glicol de etileno

B) queroseno

C) Plomo

D)metanol

E) hipoclorito de sodio

11. En un operativo de médico sin fronteras, una madre se queja de que su hijo de 3 años no es tan
grande como sus otros hijos a su edad. Un examen más detallado revela una falla generalizada del
crecimiento, pérdida de masa muscular, pero la grasa subcutánea es normal, edema prominente,
hepatomegalia, despigmentación de la piel y pelo de color arenoso o rojizo. ¿Cuál de los siguientes es
probablemente la causa contribuyente de los signos y síntomas en este paciente?

A) Beriberi

B) Kwashiorkor
C) Pellagra

D) Anemia perniciosa

E) Raquitismo

12.Femenina de 1 año de edad presenta antecedentes de diarreas e infecciones intestinales llega a


consulta por presentar pérdida extrema de peso no acorde a su edad y por falta de crecimiento, se
observan signos de desnutrición y un aumento en la circunferencia abdominal, la madre de la niña
afirma que la lacto solo por 6 meses. ¿Cuál puede ser el posible diagnostico?

A) Marasmo

B) Kwashiorkor

C) Pellagra

D) Anemia perniciosa

E )Raquitismo

1. Paciente masculino de 35 años de edad es llevado a emergencias inconsciente, al despertar el


médico nota que presenta confusión, nistagmo y oftalmoplejia. Estudios muestran deficiencia de
vitamina B1. ¿Cuál enfermedad es caracterizada por estas complicaciones?

a) Enfermedad de Wernicke

b) Neuropatía alcohólica

c) Enfermedad de la gota

d) Degeneración cerebelosa alcohólica

e) Polineuropatias

2. Un hombre de 45 años de edad, alcohólico crónico, llega a la consulta y le explica al médico que
tiene pérdida de apetito, diarrea, fatiga, dolor de cabeza y pérdida de concentración. El doctor le
explica que sus síntomas se pueden deber a un tipo de anemia que está relacionada con el
alcoholismo. El doctor le indica realizarse unos análisis para un posible diagnóstico. Varias horas más
tarde, en el análisis se ve reflejado una deficiencia de cianocobalamina (Vit. B12). ¿A qué tipo de
anemia se podría estar refiriendo el doctor?

a) Anemia falciforme

b) Anemia hemolítica

c) Anemia ferropénica

d) Anemia megaloblastica
e) Anemia aplásica

3. Juan es un estudiante de 22 años que está realizando un estudio sobre pacientes con esclerosis
múltiple y los beneficios de la Luz ultravioleta. ¿Cuál de las siguientes es cierto con relación a la luz
ultravioleta?

A. La luz ultravioleta no tiene efecto sobre el ADN

B. Las longitudes de ondas de 270 nm puede desarrollar Cáncer de piel

C. La exposición a la luz ultravioleta estimula la producción de vitamina D


D. La radiación ultravioleta NO posee gran actividad fotoeléctrica y fotoquimica

4. Paciente Ucraniano de 34 años se presenta a consulta con tos, dificultad para deglutir, ronquera e
hinchazón en el cuello. Tras varios análisis de imagen, se le encuentra un tumor tiroideo y posterior a
esto se le diagnostica Cáncer Tiroideo. En su antecedente social, el paciente explica haber estado
presente en regiones que fueron contaminadas por la Catástrofe nuclear de Chernóbil. ¿Cual de las
siguientes puede ser la causa de su padecimiento?

A.Exposición a Yodo radiactivo en su niñez


B. Radiación torácica para la enfermedad de Hodgkin

C. Consumo excesivo de Hierro

D. Radiación para tratar la espondilitis anquilosante

E. Alta ingesta de Tiamina

5.- Hombre de 47 años que trabaja en un bar se presenta a consulta y dice presentar dificultad visual,
dolor gastrointestinal. Estudios revelan que el paciente tiene necrosis de las células ganglionares de la
retina y la degeneración subsiguiente del nervio óptico y una acidosis metabólica muy elevada, media
hora después el paciente tiene convulsiones y muere.

a) Intoxicación por etilenglicol

b) Intoxicacion por plomo

c) Síndrome de tricloroetileno

d) intoxicación por metanol

6. Nino llevado a la emergencia con convulsiones repentinas, su madre refiere que su hijo salía todas
las tardes a jugar cerca de un lago al lado de la industria de zapatos. Se le realizan estudios y el niño
tiene daño al SNC, aplasia de la médula ósea, síndromes mielodisplásicos y pancitopenia. Según lo
descrito en el caso cuál de los siguientes puede producir estos síntomas?

a) Benceno
b) Tricloroetileno

c) Cloroformo

d) Plomo

7. Paciente de 25 semanas de embarazo acude a consulta por chequeo rutinario, esta refiere que se
le ha dificultado dejar de fumar ya que empezó desde muy temprana edad. El doctor le explica los
riesgos que puede manifestar por el consumo de tabaco. Cual de las siguientes manifestaciones es
más propenso a desarrollar el neonato?

A) Anencefalia.

B) Bajo peso al nacer.

C) Sifilis congenita.

D) sindrome de alcoholismo fetal.

8.Paciente femenina de 38 años llega a emergencias presentando falta de aire, dolor en el pecho,
presión arterial alta y elevadas concentraciones de Colesterol en sangre, manifiesta que fuma tabaco
desde la adolescencia y usa anticonceptivos. De acuerdo a los datos obtenidos, cuál de las siguientes
afirmaciones puede ser la causa de los síntomas?

A) Anemia Megaloblastica.

B) Infarto al miocardio.

C) Meningitis.

D) Cirrosis Criptogenica.

9- Paciente femenina de 25 años se encontraba en un festival de música electrónica, fue llevada a


emergencia por su pareja debido a que presentó convulsiones 10 minutos atrás. La paciente presenta
pupilas dilatadas, hipertensión arterial y temperatura de 39.5C. Su pareja comenta que había
consumido sustancias recreativas para disfrutar aún más el festival. cual de las siguientes fue la más
probable ingerida por la mujer?

A. LSD

B. Marihuana

C. Cocaina

D. Heroina

10- Paciente de 35 años fue encontrado inconsciente por su madre en una casa abandonada en su
vecindario, al llegar a la emergencia el paciente presenta bradicardia, hipotermia y depresión
respiratoria. Qué sustancia estaba utilizando este paciente según sus síntomas?

A. Etanol

B. Cocaina

C. Heroina

D. Flurazepam

11-Maria F. llega a tu consulta por, inflamación, cambios en el color de sus miembros inferiores y
dolor. Están calientes a la palpación. Ella tiene 23 años. Se diagnostica con una trombosis venosa
profunda. Cual de las siguientes se relaciona más con el caso?

a) Deficiencia del factor V

b) Desórdenes mieloproliferativos

c) Procedimientos ortopédicos

D) Anticonceptivos Orales

12-Una paciente de 46 años llega donde su medico con Amenorrea, hipertermia y atrofia de la vagina.
Se inicia un tratamiento de reemplazo hormonal.¿Con cuál de los siguientes se relaciona el uso
prolongado de terapia de este tipo de terapias?

a) Anemia hemolítica

b) Cáncer de mama y endometrio

c) Hepatitis Fulminante

d) Disminución de la densidad ósea

1- Un hombre de 51 años se presenta en su oficina con alfileres y sensación de aguja en ambos


brazos y piernas. Se queja de cansarse muy rápido cuando trata de subir las escaleras y hacer
recados, pero no cuando mira su programa favorito en la televisión. En el examen físico, se encuentra
una ataxia y una disminución de la vibración y el sentido de posición en sus brazos y piernas. La
evaluación posterior revela una lengua agrandada y muy roja. Cuando se realizan los laboratorios,
revelan una prueba de chequeo positiva, glóbulos rojos macrocíticos y una disminución del recuento
de Hct. ¿Cuál de las siguientes es la razón que conduce a los signos y síntomas de este paciente?

A. Anemia megaloblástica por deficiencia de cobalamina


B. Consumo crónico de alcohol

C. Deficiencia de tiamina

D. Síndrome de Wernicke-Korsakoff

E. Daño a los cuerpos mamilares

2- En un viaje de misión médica a Puerto Príncipe Haití, usted ayuda a ayudar a consultar a varios
pacientes enfermos. Una madre muy delgada le trae a su hijo de 10 años y le pregunta por qué tiene
un cuerpo muy delgado pero con una barriga grande. En el examen físico, el niño tiene un abdomen
protuberante e hinchado, una estatura demasiado corta para su edad y poca energía para un niño.
También notas varias lesiones cutáneas que parecen hiperqueratósicas y otras despigmentadas.

¿Cuál de los siguientes es más probable deficiencia nutricional en este paciente?

A. VitaminC

B. Vitamina A

C. VitaminD

D. Proteínas
 E. Calorías

3- Un jugador de fútbol de 26 años repentinamente se derrumba durante un juego y tiene un paro


cardíaco del cual los técnicos de emergencias no pueden revivirlo. Algunos de los otros jugadores en
el equipo informan que tenía un historial de abuso de diferentes drogas ilícitas. Cuando se realiza la
autopsia, el médico forense descubre una cardiomegalia moderada y ateroesclerosis marcada de las
pequeñas ramas periféricas de las arterias coronarias. El examen microscópico muestra fibrosis
irregular del miocardio. ¿Cuál de las siguientes sustancias probablemente causó estos hallazgos?

A. Heroína

B. Anfetamina

C. Lysergicacid

D. Cocaína
E. Etanol

4- Una mujer embarazada de 32 años de edad con antecedentes de hemorragia uterina y placenta
previa diagnosticada durante su ecografía del segundo trimestre, tiene una cesárea y da a luz después
de las 39 semanas de gestación a un bebé pequeño que pesa 2.250 g (5 lbs) . Aparte de que el bebé
tiene bajo peso, no presenta ninguna anomalía congénita. En este caso, ¿cuál de los siguientes
factores maternos se asocia con el bajo peso al nacer del recién nacido?

A. Fumar cigarrillos
B. Uso previo de anticonceptivos orales

C. parto prematuro

D. Deficiencia de ácido fólico

E. obesidad

5- Un excursionista inexperto de 33 años fue al Monte Gongga (ubicado en el suroeste de China) y


llegó hasta 6.575 m (altura máxima: 7.556 m) sin detenerse. Cuando un grupo lo encontró, el
excursionista yacía en el suelo, no respondía e hiperventilaba. Diez minutos después, expiró. En la
autopsia, encontraron una hipertrofia del ventrículo derecho, exudados alveolares proteináceos y
formación de membrana hialina, mientras que algunos edemas en los pulmones también eran visibles.
¿Cuál de los siguientes explica mejor este resultado?

A. Encefalopatía de altura

B. Enfermedad de montaña aguda

C. Edema pulmonar a gran altura


D. Mal de montaña crónico

E. Edema cerebral a gran altitud

6- Un hombre de 58 años con un historial de trasplante de hígado es readmitido a la sala de


emergencias, después de una semana de ser tratado por un residente de segundo año por
hiponatremia. Su esposa le explicó que tres días después del tratamiento, se presentó con
nublamiento de la conciencia, dificultad para hablar y dificultad para tragar. Después de la evaluación,
el paciente presentó problemas de pensamiento, parálisis en los brazos y las piernas, rigidez, pérdida
de la sensibilidad y pérdida de la coordinación. A los pacientes se les diagnostica mielineol pontino
central. Esto es muy probablemente causado por la administración de?

A. Administración rápida de tiamina intravenosa

B. Miranegron

C. Arabinofuranosyl citidina

D. Escitalopram

E. Administración rápida de sodio intravenoso

7- Mujer de 19 años de edad va a su ginecólogo considerando comenzar el control de la natalidad a


través de la píldora anticonceptiva oral combinada (COCP). Además del embarazo, pero no de las
ETS, los anticonceptivos orales han demostrado disminuir la incidencia de cuál de las siguientes
condiciones?

A. Colelitiasis

B. Enfermedad pélvica inflamatoria


C. Resistencia a la insulina

D. Pigmentación de la eminencia de Malar

E. Quiste y absceso de Bartholin

8- Un hombre de 25 años de edad está involucrado en un accidente automovilístico a alta velocidad y


es llevado a la sala de emergencias debido a sus lesiones. Tras el examen se determina que tiene
hemorragia interna, por lo tanto, se lo llevaron a la cirugía. El cirujano describe la fuente de la
hemorragia como una lágrima dentada con márgenes aplastados y estiramiento del tejido subyacente.
¿Qué tipo de lesión está causando el sangrado de este paciente?

A. Contusión

B. Laceración

C. Abrasión

D. laceración raspada
E. Incisión

9- David estaba haciendo skate en frente de su casa cuando accidentalmente se cayó, raspándose el
brazo derecho y la rodilla derecha en el pavimento. Como consecuencia, se eliminó la capa superior
de la epidermis. ¿Qué tipo de herida tiene David?

A. Laceración

B. Quemar

C. Contusión

D. Abrasión
E. Punción

10- Alfred es un hombre de 38 años que trabaja en el control de plagas. Presentó malestar y fatiga. Su
médico está preocupado por el posible desarrollo de encefalopatía y neuropatía periférica
caracterizada por parestesias, parálisis motoras y nefritis dolorosa. ¿Cuál de los siguientes puede
causar estos signos y síntomas en este paciente?

A. Cadmio

B. Arsénico
C. Cromo

D. Mercurio

E. Lead

11- Un hombre de 29 años con un fuerte olor a whisky es llevado al departamento de emergencias
por la policía local después de ser encontrado desmayado cerca de una estación de servicio.
Después del examen, el paciente muestra confusión mental, ataxia, motilidad ocular anormal y
polineuropatía. Las exploraciones cerebrales reflejan cambios patológicos en el diencéfalo y el tronco
encefálico. Se cree que el paciente tiene ____________ debido a una deficiencia de tiamina.

A. Encefalopatía de Wernicke
B. La psicosis de Korsakoff

C. Degeneración cerebelosa alcohólica

D. mielinólisis central pontina

12- Madeline Diaz, una estudiante universitaria de 20 años, da a luz a una niña que pesa 5 libras y 3
onzas. Al preguntar, se observa que la Sra. Díaz no recibió atención prenatal porque no estaba al tanto
de su estado de embarazo hasta que tenía 5 meses de embarazo. Cuando se le preguntó acerca de
su historia social, notó que de vez en cuando fuma marihuana y solo bebe alcohol los fines de
semana. El bebé presenta una cabeza inusualmente pequeña, un labio superior delgado y un filtrum
liso. Es probable que el bebé sufra cuál de los siguientes?

A. Síndrome de Down (trisomía21)

B. Parálisis cerebral

C. Fracaso para prosperar

D. Síndrome de alcoholismo fetal


E. Síndrome de abstinencia neonatal

1- Un pediatra está realizando un examen físico con una niña de 5 años. La madre le dice que está
preocupada porque es más baja que la mayoría de los niños de su edad. Ella te dice que la chica es
muy exigente cuando se trata de comer. Cuando examinas a la chica, notas que tiene las piernas
arqueadas, pero aquí no hay signos de daño capilar en la piel. ¿Cuál de los siguientes es el
diagnóstico más probable?

A. Escorbuto

B. Marasmo

C. Kwashiorkor

D. Raquitismo
E. Pellagra

2- Un hombre de 55 años viene a su clínica porque últimamente ha tenido dificultades para respirar. Él
le dice que camina 30 minutos todas las mañanas y que trata de no comer alimentos grasos ni nada
que sea rico en sodio. Su examen físico no es notable, excepto por la disminución de los ruidos
respiratorios en ambos pulmones. Le preguntó al paciente si fuma y él dice que sí, 2 paquetes al día
durante los últimos 30 años. Con la información proporcionada, ¿cuál es la condición más probable
que este paciente puede desarrollar debido al uso prolongado de cigarrillos?

A. Cáncer

B. enfermedad cardiovascular

C. Enfermedad metabólica (diabetes)

D. enfermedad pulmonar no maligna


E. artritis reumatoide

3. El VPH (virus del papiloma humano) es más probable que ocurra en cuál de los siguientes cánceres:

A. Cáncer de la laringe

B. Cáncer del labio, mucosa bucal y lengua


C. Cáncer del páncreas

D. Cáncer del esófago

4. La ingestión de alcohol afecta a muchos órganos y tejidos, la complicación médica más común del
alcoholismo es:

A. Hepatopatía alcohólica
B. Rabdomiólisis alcohólica aguda

C. Miocardiopatía alcohólica

D. La trombocitopenia transitoria aguda

5. Una mujer de 20 años que es sexualmente activa y sufre de endometriosis viene a su oficina de
salud para programar una cita. Ella se queja de dolor en el cuadrante superior derecho de su
abdomen. Ella describe el dolor como constante y aburrido. Desde que comenzó el dolor, se sintió
hinchada y sufrió de indigestión. El médico la diagnostica con colelitiasis. Cuál puede ser la causa de
sus síntomas clínicos:

A. Anticonceptivos orales
B. Disminución de la bilirrubina en la bilis

C. Disminución del colesterol en la bilis

D. Malformación en su conducto biliar

6. Un joven de 17 años llamado Carlos se presenta en la sala de emergencias con heridas de bala en
el abdomen. Sus heridas son profundas, penetrantes y muestran destrucción de la integridad del
tejido. Una radiografía está en camino pero en el examen notará que su herida de entrada está bien
definida y es redonda. ¿Cómo puede esperar que sea su herida de salida?

A. inexistente

B. Bien definido y redondo

C. Irregular con laceraciones estrelladas


D. Sharched agudamente

7. Un paisajista de 58 años acude al dermatólogo con una lesión cutánea en la frente. Él está muy
preocupado porque ha tenido varias lesiones similares a esta desde la infancia. Su cara y sus brazos
están cubiertos de pecas y hay un nevo asimétrico particularmente preocupante de 3 cm x 3 cm
sobre su ceja derecha. ¿Cuál de las siguientes es la causa más probable para el caso de este
paciente?

A. p53mutación

B. Reparación defectuosa de escisión de nucleótidos


C. Reparación defectuosa de ADN defectuoso

D. Exposición a radiaciones ionizantes

8. La exposición a radiación UV prolongada, específicamente UVB, está altamente asociada con el


carcinoma de células basales. Estas lesiones cutáneas se suelen caracterizar como masas nodulares
con una depresión central llena de queratina. Estudios recientes han demostrado que el daño del ADN
inducido por UV da como resultado la apoptosis de los queratinocitos. ¿Cuál de los siguientes
mecanismos describe más claramente el resultado de la irradiación UV a nivel celular?

A. Formación de enlaces fosfodiéster entre bases de guanina adyacentes en misma cadena ADN

B. Formación de dímero de citosina entrecruzada entre bases adyacentes en misma cadena ADN

C. Formación de bases desaminadas que pierden las funciones citoprotectoras de p53

D. Formación de dímeros entrecruzados entre bases de timina adyacentes en el mismo


Cadena de ADN

9. En un estudio de la Escuela de Deportes y Medicina Natural de Detroit, 8 pacientes se sumergieron


en baños de hielo (4 ° C-10 ° C) durante 30 minutos. La temperatura corporal, la frecuencia cardíaca,
la frecuencia respiratoria y la perfusión sanguínea en el cerebro se controlaron para todos los
pacientes. ¿Cuál es la respuesta normal de la vasculatura periférica?

A. La vasculatura periférica no tendrá ninguna alteración.

B. La vasculatura periférica se vasodilatará para aumentar la perfusión de las extremidades.

C. La vasculatura periférica sufrirá vasoconstricción para aumentar la perfusión del

extremidades.

D. La vasculatura periférica sufrirá vasoconstricción para aumentar el flujo sanguíneo a los


órganos principales.

10. Carla es una mujer de 22 años que ingresó en el hospital después de estar en un Festival de
Girasol durante muchas horas. Ella fue diagnosticada con un golpe de calor por el Dr. en la sala de
emergencias. ¿Cuál sería el tratamiento correcto para Carla?

A. Permita que se enfríe por sí misma

B. Administración de antipiréticos

C. Solo refrigeración externa y reemplazo de electrolitos y fluidos


D. Sumergir Carla en agua fría durante 30 minutos

11. Marcos ha estado trabajando en Camera Inc. durante los últimos 20 años. Es especialista en la
fabricación de fibras sintéticas para la empresa. El benceno es un componente principal en la
fabricación de la fibra sintética que Marcos es especialista. Ahora está retirado y presenta infecciones
recurrentes, hematomas fáciles y fatiga. ¿Los signos y síntomas que presenta Marcos son
característicos?

A. Hipertensión

B. Diabetes

C. Leucemia mieloblástica aguda


D. Insuficiencia renal

1. Fem. de 18 años se presenta con pérdida progresiva bilateral de la visión central. Se


sospecha de una neuropatía de Leber. ¿Cuál de los siguientes patrones de transmisión
es más consistente con este paciente?

MITOCONDRIAL

2. Masc. 29 años, motoconchista, sufre un accidente de tránsito que le produjo una


fractura del fémur derecho. Se realiza inmovilización de la extremidad y 2 días después
desarrolla problemas respiratorios progresivos, muere 3 días después. En el momento
de la autopsia, se tomó la siguiente imagen los pequeños vasos sanguíneos de los
pulmones y el cerebro. ¿Cuál de los siguientes es el diagnóstico más probable?

ÉMBOLOS GRASOS

3. Fem. de 40 años muere después de una larga historia de una enfermedad caracterizada
por disnea, ortopnea, hepatomegalia, distensión de las venas del cuello y edema
periférico. La superficie de corte del hígado tiene aspecto que muestra la imagen. ¿Cuál
de los siguientes trastornos es la causa más probable de estos hallazgos?

INSUFICIENCIA CARDÍACA DERECHA

4. Del caso clínico anterior, el mecanismo patológico corresponde a :


CONGESTION PASIVA CRONICA

5. Fem, de 65 años, con vida sedentaria, presenta dolor, edema y cianosis de pierna
izquierda. El cirujano vascular la lleva a cirugía. Al disecar la vena femoral izquierda se
visualiza lo que muestra la imagen. Es característico de la evolución de este evento
vascular:
EMBOLIZACIÓN

6. Fem. de 33 años, multigesta y obsesa con venas varicosas, está recibiendo consejería
del cirujano vascular para evitar la formación de trombos. ¿Cuál uso de sustancia debe
de evitar la paciente ?
ANTICONCEPTIVOS ORALES

7. Masc. 30 años , sometido a cirugía bariátrica muere por un paro cardiaco. En los
antecedentes personales patológicos tenía larga historia de cardiopatía reumática con
estenosis mitral. En la autopsia, el patólogo reporta las características de la imagen que
se refiere a:

MACROFAGOS CON HEMOSIDERINA

8. Del caso clínico anterior. ¿Estos cambios son resultados de cuál mecanismo?
CONGESTIÓN PASIVA CRÓNICA DE LOS PULMONES

9. Fem. de 14 años con anemia falciforme acude a urgencias porque está experimentando
dolor óseo intenso,y crisis vaso-oclusiva. Una tomografía computarizada abdominal
muestra evidencia, imágenes en cuña a nivel del bazo. La esplenectomía revela la
imagen, ¿cómo calificaría estas lesiones?
INFARTOS ANÉMICOS/BLANCOS

10. Fem. de 25 años con hepatoesplenomegalia masiva. Los estudios radiológicos revelan
varios defectos óseos radiolúcidos. Examen microscópico de un segmento del bazo
muestra la imagen. Esta paciente probablemente lleva mutaciones en el gen que
codifica cuál de los siguientes tipos de enzimas hidrolíticas?

GLUCOSIDASA

11. masc. 60 años ,alcohólico,es llevado al hospital con sangrado varices esofágicas. En la
autopsia, el paciente de abdomen sobresaliente contiene un gran volumen de fluido
seroso y cirrosis. ¿Cuál es el término apropiado utilizado para describir este fluido?
ASCITIS

12. Masc. 9 meses es llevado a la sala de emergencias con 3 horas de dolor abdominal
intenso y diarrea sanguinolenta. El examen físico revela un abdomen sensible .El niño
muere 24 horas después y la torsión (vólvulo) del intestino delgado se descubre en la
autopsia. El intestino delgado aparece dilatado y hemorrágico (como se muestra en la
imagen). ¿Cuál de los siguientes describe mejor estos hallazgos patológicos?
INFARTO

13. La característica citopática del virus varicela -zoster es:


INCLUSIONES INTRANUCLEARES

14. Fem. de 28 años quien cursa 2do trimestre del embarazo, la misma lleva un control
prenatal cercano ya que en su familia cuenta con 2 miembros con síndromes de Down.
Durante el último estudio sonográfico se apreció un engrosamiento del pliege nucal.
¿Cómo esperamos encontrar los siguientes marcadores en este caso?
ALFA-FETOPROTEÍNA ES BAJA

15. Masc. obrero de zona franca de calzado presenta múltiples lesiones osteolíticas en
cráneo y columna es diagnosticado con Mieloma Múltiple. ¿A cuál compuesto
carcinógeno más probablemente estuvo expuesto?
BENCENO

16. Fem. de 22 años de edad desarrolla una cara "roja" después de que se le haga una
pregunta durante la clase. ¿Cuál de las siguientes afirmaciones describe mejor esta
reacción vascular?
HIPEREMIA ACTIVA

17. Una mujer de 23 años se presenta con inicio reciente de flujo vaginal. El examen físico
revela múltiples vesículas claras en su vulva y vagina. Un frotis de material obtenido de
una de estas vesículas revela varias células gigantes multinucleadas con inclusiones
intranucleares y núcleos de vidrio esmerilado. ¿Estos cambios son probablemente por
cuál de los siguientes organismos?
VIRUS DEL HERPES SIMPLE (VHS)

18. Masc. 25 años, presenta dificultad respiratoria por más de 6 meses, como hábitos
tóxicos refiere alcohol, fumar hooka y el uso de heroína intravenosa. Se toma biopsia
del pulmón revela fibrosis intersticial del parénquima acompañada de lo que muestra la
imagen, que corresponde a:
GRANULOMAS DE TALCO

19. Una mujer de 35 años de edad, padece de síndrome antifosfolipídico; a causa de esto
desarrolla trombos en circulación arterial y venosa. Uno de estos trombos ha alcanzado
la arteria renal causa que se obstruya el aporte sanguíneo. Este fenómeno es debido a
cuál de los siguientes fenómenos en la evolución del trombo?
EMBOLIZACIÓN

20. Un hombre de 60 años de edad, originario del sur del país, presenta edema de uno de
sus miembros inferiores caracterizado por no dejar fóvea (duro) y edema del escroto,
este cuadro tiene varios meses de evolución, el cual está caracterizado por deformar el
miembro y causar incapacidad. La causa más probable de este cuadro clínico es:
OBSTRUCCIÓN LINFÁTICA

21. Un varón de 14 años presenta un síndrome caracterizado por un defecto en el tejido


conectivo con hiperelasticidad y fragilidad de la piel, hipermovilidad y dolor articular, así
como un mayor riesgo de complicaciones como disección aórtica. ¿Cuál de los
siguientes es el diagnóstico más probable?
SINDROME DE EHLERS DANLOS

22. Varón de 23 años a quien se le ha diagnosticado desde la infancia una enfermedad


hereditaria autosómica dominante caracterizada por ser muy alto, con miembros
delgados y dedos extremadamente largos, ectopia del lente y mayor riesgo de
enfermedad cardiovascular. ¿Cuál de las siguientes es la causa molecular de esta
enfermedad?
MUTACIÓN DEL GEN QUE CODIFICA FIBRINA 1 (marfan)

23. Los padres de un adolescente de 14 años acuden al pediatra con antecedente de que
lleva aproximadamente 6 meses con lesiones nodulares en la piel que han ido en
aumento. En la exploración física, estos tumores son blandos, bien definidos,del mismo
color de piel. Además, tiene nódulos pigmentados en el iris (nódulos de Lisch). El
pediatra explica que su hijo podría tener una enfermedad de origen genético. ¿Cuál de
los siguientes es el diagnóstico más probable?
SINDROME DE VON RECKLINGHAUSEN
24. El virus de Epstein- Barr (EBV) fue el primer virus vinculado con el desarrollo de
neoplasias. ¿Cuál de los siguientes cánceres está relacionado con esta infección?
LINFOMA DE BURKITT

25. Su médico le aconseja a un hombre de 53 años que se recupere de un infarto de


miocardio que tome una aspirina para "bebés" todos los días para reducir la posibilidad
de que desarrolle un segundo infarto de miocardio. La teoría detrás de este consejo es
que la aspirina disminuye la formación de trombos dentro de las arterias coronarias al
inhibir las plaquetas por la formación de qué sustancia?
TROMBOXANO

26. Un niño de 5 años fue llevado a su pediatra con antecedente de anasarca de 15 días de
evolución que ha ido en aumento. La proteinuria es evidente en la prueba de orina.
¿Cuál de las siguientes es la fisiopatología responsable del edema de este niño?
DISMINUCIÓN DE LA PRESIÓN ONCÓTICA

27. Embarazada a las 38 semanas de gestación fumadora acude a urgencias por presentar
contracciones uterinas y sangrado vaginal, se realiza cesárea obteniendo una niña
sana, pero con un peso de 2,1 kg. ¿Cuál de las siguientes podría ser una complicación
en esa chica?
MAYOR RIESGO DE DESARROLLAR OTITIS MEDIA

28. Varón de 57 años, fumador de 10 paquetes de cigarrillos al día, presenta


complicaciones de hipertensión arterial y bronquitis crónica. Acude al médico por dolor
epigástrico intenso acompañado de vómitos. El médico recomienda que deje de fumar
ya que fumar está asociado con muchas otras enfermedades. De los siguientes, ¿cuál
primer evento cree que podría estar desarrollando este paciente en el tracto
gastrointestinal?
ULCERA PEPTICA

29. Varón de 48 años que trabaja desde hace 15 años en una fábrica de baterías, es
trasladado al servicio de urgencias por presentar tos con expectoración sanguinolenta y
dificultad respiratoria severa. El paciente también informa que hay disminución en la
producción de orina, alrededor de 1 vez al día y la prueba de orina muestra proteinuria.
Días después muere el paciente. Se realizó autopsia que reveló enfisema pulmonar, así
como daño tubular y glomerular a nivel renal. Una sustancia es la responsable de estos
hallazgos. Con base en los antecedentes y los hallazgos de la autopsia, ¿cuál cree que
fue la sustancia identificada en la sangre de este paciente?
CADMIO
30. Fem. de 21 años presenta durante 7 días fiebre, adenopatías y hepatoesplenomegalia
leve y se sospecha de Mononucleosis infecciosas. ¿Cuál prueba usted indicaría para el
diagnóstico y detección de esa infección?
ANTICUERPO HETERÓFILO

31. Una mujer de 77 años desarrolló confusión aguda y debilidad en el miembro superior
izquierdo después de una artroplastia electiva de rodilla izquierda. La operación en sí se
realizó bajo un SA, transcurrió sin incidentes excepto por un episodio de hipotensión
intraoperatoria, cuando su presión arterial bajó a 90/55 y permaneció baja durante unos
30 min. Después de la operación, se observó que estaba confundida e incapaz de
mover su extremidad superior izquierda. Inicialmente, su escala de coma de Glasgow
(GCS) era 14/15 pero se deterioró rápidamente a 9/15. No tenía fiebre, su presión
arterial era de 106/60 y la frecuencia cardíaca era de 96 lpm en ritmo sinusal. Se realizó
una resonancia magnética cerebral urgente que mostró infarto en los territorios de la
arteria cerebral anterior y la arteria cerebral media (ACM). ¿Qué tipo de infarto cerebral
es este?
INFARTO EN LA ZONA FRONTERIZA

32. Un hombre de 43 años ingresó tras sufrir un accidente de tráfico. Tiene graves lesiones
en el pecho y el abdomen. Su cabeza y extremidades están mínimamente
traumatizadas. Se somete a una cirugía por una rotura esplénica. Después de ocho
horas, se vuelve hipotenso y febril, y está intubado debido a una insuficiencia
respiratoria. Desarrolla rápidamente coagulación intravascular diseminada (CID). ¿Cuál
de los siguientes es el diagnóstico más probable?
CHOQUE HIPOVOLEMICO

33. Masc. de 67 años, pescador de la provisión de Guangdong en China, presenta fiebre y


cefalea, luego de 4 días desarrolla el síndrome respiratorio agudo severo (SRAS) se
sospecha fue ocasionado por un tipo de coronavirus. En los tejidos de pulmón hay un
daño alveolar difuso, ¿cuál otro hallazgo usted esperaría encontrar en las células
sinciciales pulmonares?
CÉLULAS MULTINUCLEADAS SIN INCLUSIONES VIRALES

34. Varón de 3 meses, presenta distensión abdominal. Se realiza una ecografía que
muestra importante esplenomegalia y hepatomegalia. 3 meses después los padres
notan que el niño presenta flexión de la espalda del cuello, estrabismo y dificultad para
abrir la boca (trismo) que se intensifica con los días. Tras la realización de estudios
genéticos, el paciente fallece a los 2 años y medio de vida por complicaciones de su
enfermedad. ¿Cuál de los siguientes crees que fue el diagnóstico más probable?
ENFERMEDAD DE GAUCHER TIPO 2

35. Durante la autopsia de un hombre de 46 años que falleció cuando la motocicleta en la


que viajaba fue atropellada por un camión, se encuentra una masa roja de 1,2 cm
dentro de una rama de arteria pulmonar izquierda. En general, esta masa es gomosa,
gelatinosa y tiene una apariencia de "grasa de pollo". Los cortes histológicos revelan
que esta masa no está adherida a la pared del pulmón, y no se observan las líneas
alternas de Zahn. ¿Cuál de las siguientes afirmaciones describe mejor esta masa
intravascular?
COÁGULO DE SANGRE POST MORTEM

36. Mujer de 30 años, casada, que usa anticonceptivos orales desde hace 5 años. Su
médico explica los riesgos de usar estas hormonas sexuales, pero también habla de
algunos de los beneficios que también se han descrito. ¿Cuál de los siguientes cree que
es un beneficio de los anticonceptivos orales?
RIESGO REDUCIDO DE CÁNCER DE ENDOMETRIO

37. Varón de 63 años, fumador de 3 cajetillas de cigarrillos al día, que consulta por tos
persistente. Su médico explica que podría estar sufriendo una enfermedad pulmonar
obstructiva, pero que ser fumador también aumenta el riesgo de cáncer de pulmón y
otros cánceres. ¿Cuál de las siguientes neoplasias malignas se asocia con los
fumadores?
CANCER DE RIÑON

38. Fem. 48 años presenta sangrado vaginal anormal, especialmente post coital. Se le
realiza una biopsia que muestra un carcinoma de células escamosas del cérvix. A esta
muestra se le realiza pruebas moleculares, ¿cual es más probable que sea el agente
identificado?
VIRUS DEL PAPILOMA HUMANO TIPO 16 o 18

39. Varón de 25 años diagnosticado de trisomía 21, presenta múltiples hematomas


asociados a trombocitopenia, palidez generalizada de la piel, evidenciándose en el
hemograma que presenta anemia severa y disminución de glóbulos blancos. Teniendo
en cuenta las enfermedades asociadas con estos pacientes, ¿cuál de las siguientes
enfermedades cree que podría ser la causa de la afección actual?
LEUCEMIA AGUDA

40. Fem. 24 años muere después del parto. Revisión de la historia clínica revela que se
había enfermado gravemente con disnea ,hipotensión y convulsiones, y una radiografía
de tórax había demostrado evidencia de edema pulmonar. En la autopsia, los vasos
sanguíneos en los pulmones, lo que muestra la imagen. Esto fue todo seguido de
trombosis generalizada .Los cambios que se encontraron dentro de múltiples vasos
sanguíneos lo más probable es embolización de :
LÍQUIDO AMNIÓTICO
Genetica

1-A cuatro años de edad, presenta las niñas para un examen físico en edad preescolar. El niño
tiene una circunferencia de la cabeza pequeña, labio superior delgado, y bajo puente de la
nariz. Ella muestra la evidencia de los trastornos mentales leves retraso. Sus padres decir que
ella es a menudo "emocional".¿Cuál de las siguientes causas maternas de la mayoría de los
defectos de nacimiento probablemente explica estos hallazgos clínico-patológicos?

(A) El abuso de alcohol

2 -Un niño de 12 meses de edad, es llevado a la sala de emergencias para examen de su


brazo derecho tras una caída en su casa.El examen radiológico de la extremidad revela una
fractura reciente del cúbito derecho y la evidencia de fracturas de curación adicionales. El niño
se nota que tienen escleras azules. Este paciente más probable que porta una mutación en un
gen que codifica la que de la después de las proteínas?

(A) El colágeno- osteogenesis imperfecta

3 - mujer de 28 años da a luz a un muerto con un defecto del tubo neural severa (recién nacido
se muestra en la imagen). Este defecto de nacimiento fue causado por un error de la
morfogénesis que ocurrió en cuál de las siguientes etapas de desarrollo después de la
fecundación?

20 a 40 dias.

4-Un hombre de 20 años de edad, es examinado por un médico de familia nueva que descubre
numerosas manchas pigmentadas de la piel y tumores pediculados en el pecho. La biopsia de
un tumor describe un tumor benigno derivado de las células de Schwann. Ni el padre ni la
madre del paciente muestra signos de esta enfermedad. Este paciente más probable lleva una
mutación en un gen que codifica la que de las siguientes proteínas?

(B) GTPasa proteína activadora-neurofibromatosis tipo 1-cromosoma 17

5-El paciente descrito en la pregunta 4 se encuentra en mayor riesgo de desarrollar cuál de las
siguientes neoplasias malignas?

(C) neurofi brosarcoma


6- Una mujer embarazada de 25 años de edad, a las 16 semanas de gestación, las visitas su
obstetra. Una prueba de detección sugiere la posibilidad de una defecto del tubo neural en el
feto. Un examen de ultrasonido muestra un defecto de 3 cm del tubo neural en la espina
torácica. La prueba de tamizaje que se administró a la madre medido los niveles séricos de
cuál de las siguientes proteínas?

(C) La alfa-fetoproteína

7-Un hombre de 25 años de edad se presenta para un examen físico de rutina. El paciente es
alto (6 pies, 5 pulgadas) de largo y tiene ngers inalámbrica a internet (se muestra en la
imagen). Un año después, sufre un aneurisma disecante. Este paciente más probable lleva una
mutación en un gen que codifica cuál de las siguientes proteínas.

fibrilina- sindrome de marfan

8-La enfermedad genética que se encuentran en el paciente descrito en la pregunta 7, que


sigue de los siguientes patrones de herencia?

autosómica dominante- sindrome de marfan-

9-Un niño de 12 meses de edad, muestra una debilidad progresiva, deterioro mental y pérdida
de la visión. Los estudios de laboratorio demuestran disminución de la actividad de la
hexosaminidasa A. El niño con el tiempo se vuelve ciego y muere a los 3 años de edad. ¿Cuál
de las siguientes opciones describe mejor la patogénesis de la degeneración neuronal en este
paciente?

(A) La acumulación de sustrato no metabolizado- Tay- Sachs disease

10-Si los padres del niño se describe en la pregunta 9 con un total de cuatro hijos y dos hijas, a
continuación, en promedio, ¿cuántos de sus hijos se puede esperar para ser asintomática (es
decir, en silencio) portadores de esta mutación genética?

(B) Tres niños- Tay- Sachs disease

11- Un niño de 4 años de edad, está ingresado en el hospital con neumonía y dificultad
respiratoria. Las enfermeras informan de que los movimientos del niño son grasas del intestino
y tienen un olor acre. Una prueba de sweatchloride es positivo. ¿Cuál de los siguientes
mecanismos de la enfermedad es la causa más probable de la esteatorrea en este niño?

(C) La falta de secreción de enzimas pancreáticas- fibrosis cistica


12 - El paciente descrito en la pregunta 11 lleva a mutaciones en el gen que codifica cuál de los
siguientes tipos de proteínas?

(A) de membrana de canales de iones- fibrosis cistica

13 - Un niño de 10 años de edad se presenta con xantomas en las superficies extensoras de


los antebrazos. Los estudios de laboratorio demuestran un colesterol total en suero de 820 mg /
dl. La madre del niño y el abuelo materno también tienen el colesterol elevado. Este paciente
tiene más probabilidades de mutaciones en el gen que codifica cuál de las siguientes proteínas
implicadas en el metabolismo de lípidos?

(D) del receptor de lipoproteína de baja densidad-(LDL)- familia hipercolesteronemia-


cromosoma19

14 -Un niño de 10 meses de edad que fue adoptado de un orfanato en Europa del Este se
presenta para un examen físico. Sus padres creen que no está cumpliendo los hitos del
desarrollo. El niño es de piel blanca y tiene el pelo rubio. En el examen físico, el paciente se
observó a tener un "ratón" el olor. Los estudios de laboratorio demuestran un error innato del
metabolismo de los aminoácidos. Para prevenir el retraso mental, este paciente debe ser
colocado en una dieta especial que carece de cuál de los siguientes aminoácidos esenciales?

(C) fenilalanina- autonomica recesiva

15 - ¿Cuál de las siguientes opciones describe mejor la patogénesis de retraso mental en el


paciente descrito en la pregunta 14?

(A) La acumulación de sustrato no metabolizado- PKU

16 -Un niño de 4 años de edad, se encuentra que tiene la piel muy flexible. Sus padres en
cuenta que las equimosis con gran facilidad. Sus articulaciones pueden ser hiperextensión. Los
estudios bioquímicos demuestran una defi ciencia de la lisil hidroxilasa. El examen
ultraestructural de una biopsia de piel de la paciente lo más probable es revelar las
anormalidades asociadas con cuál de las siguientes células / tejidos componentes?

Fibras de colageno - Ehlers- danlos sindrome- autonomica dominante

17 -Una mujer de 25 años de edad, se queja de dolor en los huesos recurrentes y cada vez
mayor la circunferencia abdominal. El examen físico revela hepatoesplenomegalia masiva. Los
estudios radiológicos revelan varios defectos óseos radiolúcidas. Una biopsia de médula ósea
revela células alargadas con una apariencia que recuerda fi Brillar de "un pañuelo de papel
arrugado." El examen microscópico de una muestra de la esplenectomía se muestra. Este
paciente más probable lleva mutaciones en el gen que codifica cuál de los siguientes tipos de
enzimas hidrolíticas?

(B) glucosidasa - gaucher disease

18 - ¿Cuál de las siguientes opciones describe mejor la patogénesis de la


hepatoesplenomegalia y dolor óseo en el paciente descrito en la pregunta 17?

(A) La acumulación de sustrato no metabolizado- gaucher disease

19 - Un recién nacido nace con la disfunción motora severa en las extremidades inferiores. Los
estudios radiológicos muestran que los cuerpos vertebrales en la región lumbar carecen de
arcos posteriores. Los defectos vertebrales están cubiertos por una membrana delgada. El
espacio debajo de la membrana que contiene una masa de tejido que se compone de las
meninges y la médula espinal. Los padres piden información sobre los riesgos de defectos de
nacimiento similares en su futura descendencia. Usted menciona que la suplementación
dietética de la dieta materna se ha demostrado que puede reducir la incidencia de defectos del
tubo neural. ¿Qué es esta sustancia?

(A) de ácido fólico- espina bífida

20 -Los padres de un bebé con labio leporino y paladar hendido (bebé aparecen en la imagen)
visitar a un consejero en genética para discutir la posibilidad de que un defecto de nacimiento
similar ocurrirá en su futura descendencia. Además de la exposición teratógeno y la herencia
multifactorial, cuál de las siguientes es una importante causa de este error de morfogénesis?

(C) anormalidad cromosómica structural-labio leporino- b12

21 - Un niño de 4 años de edad, es llevado al médico por sus padres porque se cansa
fácilmente. El examen físico revela debilidad en las cinturas pélvica y escapular y de la
ampliación de los músculos de la pantorrilla del niño. Los niveles séricos de creatina quinasa
son elevados. Una biopsia del músculo de la pantorrilla muestra una marcada variación en
tamaño y forma de las fibras musculares. Hay focos de necrosis muscular fi bra, con
myophagocytosis, la regeneración de fi bros, y fi brosis. Ensayos de diagnóstico molecular
mostraría más probable es que alteraciones en la longitud de la transcripción primaria para que
de los siguientes asociados músculo-proteínas?

(C) La distrofina- sindrome distrofia muscular duchenne

22 - ¿Cuál será la causa probable de la muerte en el paciente descrito en la pregunta 21?


(A) Miocardiopatía- duchenne

23 - Un hombre de 22 años de edad, se queja de su incapacidad para concebir un hijo. En el


examen físico, el paciente se observó a ser alto (6 pies, 5 pulgadas) y las exposiciones
ginecomastia y atrofia testicular. Los estudios de laboratorio demuestran el aumento de los
niveles séricos de hormona folículo-estimulante. Los estudios citogenéticos revelan una
anomalía cromosómica. ¿Cuál es la causa más común de anomalía cromosómica de este
paciente?

(B) no disyunción meiótica- klinefelter 47xxy

24-Una mujer embarazada de 35 años luz a un bebé prematuro a las 28 semanas de


gestación. Poco después del nacimiento, el recién nacido se queda sin aliento, con retracción
intercostal y aring nasal baja durante la respiración. El recién nacido es colocado en un
respirador artificial,

pero se muere de la insufi ciencia respiratoria e intraventricular hemorragia. El examen


microscópico de los pulmones a autopsia se muestra. El material que recubre eosinofílica los
espacios de aire representa una acumulación de cuál de las siguientes proteínas?

(C) de fibrina - sindrome de distress respiratorio- deficiencia del sucfaltante- membrana hialina

25 - Si el recién nacido se describe en la pregunta 24 había sobrevivido, ¿cuál de las siguientes


opciones sería la complicación más probable en relación a la anoxia y la acidosis?

(C) La enterocolitis necrotizante- sindrome distress respiratorio

26 - Una joven de 16 años de edad, se queja de que no ha empezado a menstruar al igual que
otras niñas de su edad. El paciente es corta (4 pies, 11 pulgadas) y tiene un cuello grueso-
palmeados. El examen físico revela pezones ampliamente espaciados y desarrollo de los senos
pobres. Si la enfermedad genética de este paciente fue causada por la no disyunción durante la
mitosis de una célula somática en las primeras etapas de embriogénesis, cuál de los siguientes
es más probable cariotipo del paciente?

B.45x/46'xx- nodisjuncion - mosaico- turner-

27-Una mujer de 34 años de edad, en su segundo embarazo ofrece un recién nacido femenino
severo edema generalizado e ictericia. Un CSC muestra del recién nacido anemia hemolítica. A
realizar a continuación de la madre y el recién nacido revelan una incompatibilidad Rh.Paso
transplacentario de cuál de las siguientes proteínas es la causa principal de anasarca y la
ictericia en este recién nacido?
(C) IgG- eritoblastocis fetal- anemia hemolitica

28-Los padres de un niño de 2 años de edad, con hyposadias (uretra se abre en la cara ventral
del pene) una visita a un especialista en genética para discutir las posibilidades de que un
defecto de nacimiento similares tendrán lugar en su futura descendencia. Este defecto de
nacimiento muestra cuál de los siguientes patrones de herencia?

(C) multifactorial- hiposAdias-

29 - Una mujer de 42 años da a luz a un recién nacido con múltiples anomalías congénitas.
Hallazgos físicos incluyen una baja en el perfi l cara, ojos rasgados, epicanto, Brushfi puntos
ELD, nariz corta, cuello corto, orejas displásicas, clinodactilia, una lengua larga que sobresale,
y un soplo en el corazón pronunciado. ¿Cuál es la causa más común de esta enfermedad de
nacimiento en el desarrollo?

(E) no disyunción- sindrome 21-leucemia

30 - Como adulto, el cerebro del paciente que se describe en la pregunta 29 se muestran los
cambios histopatológicos que se observan en los pacientes con cuál de las siguientes
enfermedades neurológicas?

(A) la enfermedad de Alzheimer- trisomia 21

31 - Un hombre de 50 años de edad, con antecedentes de diabetes mellitus tipo 2 se


pregunta sobre las posibilidades de que sus hijos van a heredar este trastorno metabólico. El
paciente se le dice que tiene una enfermedad genética que muestra cuál de los siguientes
patrones de herencia?

(B) multifactorial- diabetes mellitus

32 - Un hombre de 25 años de edad, con antecedentes de autismo y retraso mental es visto


por un especialista en genética. El hombre tiene rasgos faciales toscos, un incremento del
perímetro cefálico y macro orquidismo. Su tío materno se ven igualmente afectados. Después
de una evaluación adicional, un diagnóstico de síndrome de X frágil se vuelve. ¿Cuál es la
causa más probable de la enfermedad genética de este paciente?

(C) La expansión de repetición de trinucleótidos-retardacion mental- sindrome x frágil

33-Un hombre de 28 años de edad, presenta a la sala de emergencia de 1 hora después de


experimentar dolor torácico opresivo retroesternal. Estudios de laboratorio y rm ECG confianza
el diagnóstico de infarto agudo de miocardio. El paciente fallece 24 horas después de la
arritmia cardiaca. Este paciente tenía muy probablemente cuál de los siguientes enfermedades
genéticas?

(C) La hipercolesterolemia familiar-autonómica dominante


34-Un niño de 5 años de edad se presenta con una erupción maculopapular. En el examen
físico, la erupción afecta a las palmas y plantas. Las grietas y fi suras se observan alrededor de
la boca y el ano. Hay evidencia de fondo de ojo de la queratitis intersticial. Leve
hepatoesplenomegalia está presente. Los huesos tibial anterior exhiben una curvatura
hacia el exterior. ¿Cuál es la etiología más probable de estos hallazgos clínico-patológicos?

(C) la sífilis-sindrome torchh- no va

35 - Un niño de 3 años de edad, muere en un accidente automovilístico. En la autopsia, el


pulmón derecho está marcadamente reducido. La disección demuestra que el derecho de los
bronquios del tallo principal termina ciegamente en el tejido indescriptible compuesto por
conductos rudimentarios y tejido conectivo. Este hallazgo fi representa un ejemplo de cuál de
los siguientes errores de la morfogénesis?

(A) Aplasia-pulmón

36-La madre de un niño recién nacido se alarmó de que su bebé regurgita en cada toma. Un
examen endoscópico del esófago revela que el niño está casi completamente obstruido. Este
hallazgo fi representa un ejemplo de cuál de los siguientes errores de la morfogénesis?

(C) Atresia- formación incompleta del lumen "esófago"

37-Una mujer de 87 años de edad, muere tranquilamente mientras dormía. En la autopsia, el


resto del tejido pancreático es identifi cado en la pared del esófago inferior. Este hallazgo fi
representa un ejemplo de cuál de los siguientes tumores congénitos como las condiciones?

(A) coristoma-resto del tejido pancratico

38 - Una mujer embarazada de 30 años de edad, visita a su obstetra para la atención prenatal y
del parto final. Los pacientes voluntarios que dos de sus tres hijos tenían "ictericia" en el
nacimiento. Su hija menor había sido severamente ictericia y había recibido dos de sangre
transfusiones. Las pruebas prenatales de laboratorio indican que la madre es el tipo de sangre
O Rh negativo, mientras que su esposo es el tipo de sangre A, Rh positivo. Las muestras de
obstetras fl uido amniótico a las 36 semanas de gestación para determinar si el feto es lo
suficientemente maduro para el parto prematuro. El análisis cuantitativo de cuál de los
siguientes fue más probable es utilizado como un indicador de la madurez pulmonar del feto?

(D) Lecitina

39 -El paciente descrito en la pregunta 38 ofrece a una mujer a las 37 semanas de gestación,
con evidencia de edema generalizado severa (recién nacido se muestra en la imagen). El bebé
recibe transfusiones de intercambio con las células Rh negativo, pero muere posteriormente.
¿Cuál de las siguientes opciones describe mejor la patogénesis de la anasarca en este bebé?

(B) La insuficiencia cardíaca congestive --eritoblastocis fetal

40 - Una mujer de 18 años de edad, ofrece un recién nacido a raíz de una mano de obra
masculina difi culta y el parto (distocia). El bebé se ve fuerte al nacer (puntaje de Apgar = 9),
pero un cefalohematoma es evidente 2 horas más tarde. Una tomografía computarizada de la
cabeza del bebé muestra hemorragia subperióstica más de uno de los huesos craneales. ¿Cuál
es el resultado más probable de esta complicación del parto y el alumbramiento?

(D) La resolución espontánea

41 - Una mujer de 42 años de edad en su tercer embarazo ofrece un recién nacido femenino a
las 30 semanas de gestación. El bebé se desarrolla ictericia dentro de 2 días. La bilirrubina no
conjugada suero es de 15 mg / dl. ¿Cuál de las siguientes es la complicación más grave de la
hiperbilirrubinemia no se trata en este recién nacido?

(C) la encefalopatía

42 - Una mujer de 27 años de edad, presenta una prueba de embarazo. Recuerda beber
mucho durante la semana en la que ella pudo haber concebido. ¿Cuál es la consecuencia más
probable de la exposición tóxica a las embrión en los primeros meses (previo a la implantación)
el desarrollo?

B. letalidad embrionaria

También podría gustarte